تغطية شاملة

المادة المظلمة تلقي دليلاً / كلارا موسكوفيتش

وبعد عقود من النتائج السلبية، وجد الفيزيائيون أخيرا حلا

المجرات المتصادمة في العنقود أوبل 3827. بإذن من المرصد الأوروبي الجنوبي (ESO)، وكالة ناسا ووكالة الفضاء الأوروبية/تلسكوب هابل الفضائي
المجرات المتصادمة في العنقود أوبل 3827. بإذن من المرصد الأوروبي الجنوبي (ESO)، وكالة ناسا ووكالة الفضاء الأوروبية/تلسكوب هابل الفضائي

هناك شيء ما في مكان ما في الكون. لا يمكننا رؤيته، ولا يمكننا لمسه، ونعلم بوجوده فقط بسبب قوة الجاذبية التي تمارسها على الأجسام الموجودة في الكون. لعقود من الزمن، كانت قصة المادة المظلمة عبارة عن اكتشاف تلو الآخر حول ماهية هذه المادة الغامضة، وهي تصفية تدريجية للاحتمالات جعلت علماء الفيزياء يشعرون بالتوتر المتزايد. ماذا سيحدث عند حذف آخر مرشح من القائمة؟ هل سيكون مصيرنا محتومًا، دون أن نتمكن أبدًا من إلقاء نظرة على طبيعة المادة التي تساهم بحوالي 25% من إجمالي كتلة الكون؟

تُظهر البقع الزرقاء ضوءًا منحنيًا، يستخدمه علماء الفلك لقياس أربع مجرات مندمجة.

اتخذ هذا السيناريو الكئيب منعطفًا مفعمًا بالأمل في أوائل ربيع عام 2015. فقد كشف الباحثون عن أحد أكثر القرائن إثارة للاهتمام منذ سنوات: علامة على وجود قوة جديدة قد تسمح للمادة المظلمة "بالتحدث" مع نفسها. ستساعد هذه الرؤية في تفسير نوع الجسيمات التي ربما تتكون منها المادة المظلمة.

جاء هذا الدليل من ملاحظات زاوية من الكون تعرف باسم أوبل 3827. حدد علماء الفلك مؤخرًا موقع المادة المظلمة ضمن أربع مجرات متصادمة في هذه المجموعة باستخدام ظاهرة تعرف باسم انحناء الجاذبية (انحناء الضوء أثناء مروره بالقرب من الأجسام الضخمة). ). كشفت الملاحظات التي تم إجراؤها باستخدام تلسكوب هابل الفضائي و"التلسكوب الكبير جدًا" في تشيلي أن المادة المظلمة المحيطة بواحدة على الأقل من هذه المجرات تتخلف خلف المادة العادية هناك. تشير الملاحظة إلى أن جسيمات المادة المظلمة تتفاعل مع بعضها البعض وتبطئ نفسها: وهي ظاهرة لم يسبق لها مثيل من قبل.

ويتوقع فريق من علماء الفلك بقيادة ريتشارد ماسي من جامعة دورهام في إنجلترا أنه بما أن التفاعلات لم تؤثر على المادة الطبيعية، فلا بد أنها حدثت من خلال قوة أخرى غير الجاذبية، وهي القوة التي تؤثر فقط على المادة المظلمة. فتبادل "الفوتونات المظلمة" على سبيل المثال قد يخلق هذه القوة. قد يكون مثل هذا الوضع مشابهًا للطريقة التي تتفاعل بها البروتونات العادية مع بعضها البعض من خلال القوة الكهرومغناطيسية: عندما يقترب بروتونان من بعضهما البعض، يطلق كل منهما فوتونًا حاملًا للقوة الكهرومغناطيسية، ويقوم الآخر بامتصاصها. تنقل هذه التبادلات الزخم، مما يؤدي إلى انفصال البروتونين عن بعضهما البعض.

أثار هذا الخبر الفيزيائيين وألهمهم للبحث عن إجابات. يقول الفيزيائي نيل وينر من جامعة نيويورك، الذي لم يشارك في البحث: "إذا صمد هذا الاكتشاف، فهو أكثر بكثير من مجرد أخبار مثيرة". السيناريو الذي يتضمن الفوتونات المظلمة هو تعديل للمفهوم الأساسي والمقبول للمادة المظلمة باعتبارها تتكون من نوع واحد فقط من الجسيمات، والذي يُطلق عليه عادةً "جسيم ضخم متفاعل ضعيف"، أو WIMP ("ضعيف"، باللغة الإنجليزية ). لكن فكرة أن المادة المظلمة مصحوبة بفوتونات مظلمة وتفاعلات غريبة قد تساعد في حل بعض المشكلات التي أثيرت من خلال وصف المادة المظلمة باستخدام نوع واحد من WIMP، مثل سبب كون مراكز المجرات أقل كثافة من المتوقع.

قد يساعد هذا المفهوم أيضًا الفيزيائيين على تضييق نطاق قائمة المواد المظلمة المرشحة إلى حد كبير. يقول وينر: «على الرغم من أننا نمتلك أدلة على وجود المادة المظلمة من مجموعة كبيرة ومتنوعة من المصادر، إلا أنه ليس لدينا في الوقت الحاضر أي شيء يحافظ بشكل واضح على أي شيء يتجاوز تفاعلاتها الجاذبية. إذا اتضح أن لديها تفاعلات ذاتية على هذا المستوى، فيمكننا القضاء على عدد كبير من النماذج" التي تصف ماهية المادة المظلمة. على وجه الخصوص، قد تتعارض هذه النتيجة، التي نُشرت على الإنترنت في أبريل/نيسان ومطبوعة في يونيو/حزيران 2015 في مجلة الإشعارات الشهرية للجمعية الفلكية الملكية، مع العديد من الإصدارات الشائعة من الفرضية القائلة بأن المادة المظلمة هي جسيم تنبأت به نظرية التناظر الفائق. هذه النظرية، وهي فكرة مغرية تحاول تفسير العديد من الألغاز في الفيزياء، مثل سبب انخفاض كتلة بوزون هيغز، تفترض أن هناك جسيمات أخرى في الكون لم يتم اكتشافها بعد. ومع ذلك، إذا كانت إحدى هذه الجسيمات (والتي قد تكون WIMP) مسؤولة عن المادة المظلمة، فإن معظم إصدارات النظرية لا تتنبأ بالتفاعلات الذاتية.

يقول مؤلفو الورقة الآخرون إنه من السابق لأوانه استبعاد التفسير اليومي لملاحظاتهم. على سبيل المثال، من الممكن أن المادة المظلمة الموجودة خارج المجرات المتصادمة، ولكن ضمن خط البصر من الأرض، قد تساهم في قوة الجاذبية. يقول عضو الفريق ديفيد هارفي من المعهد السويسري للتكنولوجيا في لوزان: "أحد التحذيرات في هذه الدراسة الجديدة هو أنها مجرد عظمة واحدة". "نحن لا نعرف ما لا نعرفه، وهذه العوامل المجهولة قد تؤثر على النتيجة." بالإضافة إلى ذلك، لم تكشف عمليات البحث السابقة في مجموعات أخرى عن علامات تفاعل المادة المظلمة مع نفسها، بما في ذلك دراسة نشرت في مارس 2015 في مجلة ساينس، برئاسة هارفي، الذي قام بتحليل 72 تصادمًا لعناقيد المجرات، بدلاً من المجرات الفردية. ومع ذلك، فإن العناقيد تتصادم بشكل أسرع من المجرات، لذلك هناك وقت أقل للمادة المظلمة للتفاعل والتأخر، وبالتالي فإن النتيجتين لا يستبعد أحدهما الآخر.

إذا اتضح أن الملاحظات الأخيرة لا تعكس قوى أو تفاعلات جديدة للمادة المظلمة، فسيصبح أوبل 3827 مثالًا آخر لما لا تمثله المادة المظلمة. وفي هذه الأثناء، يستمر البحث عن جزيئاته في أجهزة الكشف تحت الأرض للكشف عن الطين. بالإضافة إلى ذلك، لم تظهر المادة المظلمة بعد في مصادم الهادرونات الكبير (LHC) التابع لـ CERN. ويأمل العلماء أن تتغير هذه الاتجاهات قريبًا: فقد استأنف المسرع عمله في أبريل 2015، بأعلى طاقات وصل إليها على الإطلاق، وأصبحت أجهزة الكشف الآن حساسة بشكل لا يصدق. يقول هارفي: "لقد كانت المادة المظلمة بعيدة المنال حتى الآن، لكننا لم نحصل أبدًا على البيانات التي تأتي في طريقنا". "في رأيي، إما الآن أو أبدا."

تعليقات 235

  1. إسرائيل شابيرا
    صدقوني، قرأت عدة كتب حول هذا الموضوع. لقد فهمتهم. ليس انت. من الممكن أن تفهم صفحة Gemara التي لست كذلك. لديك موهبة غير عادية في تناول موضوع يكفي أن تكتب عنه فقرة، اثنان على الأكثر وأنت. خذ قلمًا وأفرغ كل الحبر. ريتشارد فاينمان الذي حصل على جائزة نوبل لن يفهمك. من الممكن الكتابة عن موضوع في العالم معقد بقدر ما قد يكون بطريقة موجزة وواقعية. أنت لا تعرف كيف تفعل ذلك. حبل. يبدو أن روح ما بعد الحداثة تحل عليك. وبهذا أنهي جدالي معك.

  2. إريك وإسرائيل شابيرا
    ليس لدي أي ادعاءات، أنت تفعل، وهذه هي الطريقة التي تبدو بها. تتجول في المياه الصفرية للفوتونات والكواركات وتفكر أنك تفهم. أنت مخطئ. ستظل بوزونات هويجنز تدفع عقلك إلى الجنون، على الرغم من أنه ليس من المؤكد أن الأمر واضح. لقد شربت الكثير من الفودكا.

  3. حسنًا يا جوريلون، ما هو الإنجاز العلمي الذي وصلت إليه؟ تقشير الموز باستخدام الذيل؟ وما الجائزة التي حصلت عليها؟ كأس جوز الهند؟

    ربما يمكنك أن تشرح لنا كيف تجذب الأرض القمر؟ بحبل طرزان؟

    الإنسانية تحبس أنفاسها.

    وكذلك حيوانات الغابة.

    الى الأمام!

  4. عزيزي الغوريلا
    نحن جميعًا قردة بشرية نحاول فهم الأشياء وتبادل الخبرات. وبعد أن نقدم جميعنا شرحنا بدورنا، نفهم المادة بشكل أفضل. ومن كل تعليمي تعلمت!
    اسبوع جيد
    وردة صغيرة من الشجرة
    يهودا

  5. لجميع المستجيبين بجميع أنواعهم. ربما التوقف عن لعب دور الفلاسفة على عشرة سنتات. لم يتوصل أحد منكم إلى أي اختراق علمي. أنت لم تحصل ولن تحصل على جائزة نوبل. أنت تتحرك في دوائر مثل كلب يطارد ذيله. ربما افعل شيئًا أكثر إنتاجية مثل قراءة كتاب. وكما يقولون، الملل هو أم كل الخطايا. من عز سنيني أشفقت عليك.

  6. تفجير بيضة، أي إله تقصد؟
    إلى بوذا؟ إلى زيوس؟ إلى أودين؟ ربما في سبيل الله؟
    خلال تاريخ البشرية المكتوب، آمن الناس بحوالي 1000 إله مختلف. اختر واحدة وسأكون سعيدًا إذا قمت بشرح اختيارك.

  7. إلبينتزو,
    ارجو قبول اعتذارى.
    منذ بضع سنوات حاولت مشاركة أفكاري هنا على موقع العلوم عن طريق التعليقات والمحادثات مع الأصدقاء. لقد بدأت من نقطة انطلاق "الكون من لا شيء" وحاولت إظهار أن "اللا شيء" هو كيان (بتعبير أدق، العدم) له خاصية واحدة نشطة - النفي. حاولت أن أشرح كيف يتم خلق شيء ما من لا شيء دون تدخل خارجي (نوع الإله الذي ذكره تامبالتسو). حتى أنني استعينت بالتورية (لا-لا تعطي نعم، ولكن نعم-نعم لا تعطي لا). ومع ذلك، كنت أواجه دائمًا جدارًا من سوء الفهم من نوع "لكن أين تسخير الحصان"، مثل "ما كتلة الجسيم الذي يخلق كتلة؟". لقد زعمت أنه بما أن الرقم "الخيالي" i هو ضرورة الواقع، فإن هناك أيضًا قيمة حقيقة منطقية يعطي تطبيقها على نفسها خطأً؛ لا نعرفها في الواقع اليومي، لكن وجودها يحل مشكلة عدم التماثل في المنطق ويبين أن قيمتي الحقيقة المعروفتين، TRUE & FALSE، تشتقان منها، كما أنها حل لسؤال لماذا الفضاء الذي نعمل فيه فيه ثلاثي الأبعاد.
    للحظة اعتقدت أنك قد زرت هنا بالفعل ...

  8. يوفال
    الشيء الوحيد الذي لا يحتاج إلى شرح في هذا العالم هو الله.
    لقد شرح نفسه جيدًا بالفعل.
    علينا فقط أن نحاول فهم ما كان يقصده.
    ؟؟؟؟

  9. اليوبيل,

    1. أنا لا أمزح معك.

    2. ليس لدي أي فكرة عن هويتك ولم أقرأ أبدًا أي شيء كتبته (إلا إذا قمت بنشره على JHEP أو شيء من هذا القبيل، على الرغم من أنني أعتقد أنني سأتذكر اسمك حينها).

    3. لست الوحيد الذي أراد تعريف البنية الرياضية المجردة التي من خصائصها الرياضية وحدها تنشأ جميع الظواهر الفيزيائية. لقد فعلوا ذلك من قبلك. في الواقع، توراة سدوت هي بالضبط مثل هذه التوراة. ليس هناك جسيم فيه، ولا تعريف للكتلة أو القوة. كل ما هناك هو الفضاء المتجه (Fock Space)، الذي يتم تعريفه بسلسلة من النسخ ذات خصائص تناظر معينة، والفراغ (بالمعنى الرياضي، وليس بمعنى الفراغ)، وما إلى ذلك. ومن هذه التعريفات نحصل على الجسيمات، ونحصل على مفهوم القوى والثبات، ونحصل على الكتلة، وما إلى ذلك.

  10. على الأقل بالنسبة لي، يبدو الأمر مثل هاملت.
    وأنا لا أتحدث عن المسرحية.
    أنا أتحدث عن الطعام المصنوع من البيض.
    لماذا لا نذهب أبعد من ذلك؟
    - الله اخترع وحدد الجزيئات؟

  11. إلبينتزو
    هل تمزح معي. صحيح؟
    لا أعرف من الذي يختبئ خلف لقبك (لا أستطيع إلا أن أخمن)، ولكن أظن أنك قرأت فصولاً من النموذج الخاص بي 🙂
    وبالفعل، أبدأ ببنية رياضية، تفتقر إلى أي خصائص فيزيائية، ومن خلال التحليل الرياضي وحده أصل إلى هياكل معقدة أعزوها إلى جزيئات المادة المظلمة.

  12. بسبب ظاهرة الاضمحلال المرتبطة بالكتلة (التي أثبتتها ملاحظة إدينجتون)، افترض الفيزيائيون أن الموجات الكهرومغناطيسية في الجسيمات (الفوتونات) تتمتع بالكتلة. وبما أنه لم يتم قياس مثل هذه الكتلة، فقد اخترعوا مفهوم "كتلة الحركة".
    إن ظاهرة الشيخوخة هي مجرد واحدة من عدة ظواهر مرتبطة بالكتلة. اثنان آخران معروفان هما الجاذبية والمثابرة. هذه ثلاث خصائص مختلفة بشكل أساسي، وأنا مهتم بمعرفة الآلية التي تنتجها والظواهر الأخرى المحتملة.

  13. اليوبيل,

    لماذا لا نتقدم خطوة أخرى؟ أترك لكم الجزيئات. دعونا نحدد فقط بنية رياضية، خالية من أي خصائص فيزيائية، والتي وفقا لبنيتها (أساسا التماثلات ولكن ليس فقط) ستحدد الجسيمات الموجودة. وهذا يعني أنه يمكنك النظر إلى الفيزياء حيث الجسيمات ليست أشياء أولية على الإطلاق. كيف هذا الصوت بالنسبة لك؟

  14. يودا، إسرائيل
    يتمتع بوزون هيغز بكتلة نتيجة للتفاعلات داخل مجال هيغز (إذا فهمت ذلك بشكل صحيح).
    ولا تنس أن هناك ما يسمى "الطاقة المظلمة". ربما "يعطي" كتلة لجزيئات المادة المظلمة؟ أو الجاذبية تدفع الجسيمات؟
    اه... انا اصلا عندي اختلافات في الضغط في دماغي..

  15. يودا

    أنا لست أذكى منك، ولكني سألت هذا السؤال. وفقا ل "كسر التماثل" إليك.

    لكن في نموذج ميكانيكي بسيط مثل الدفع، لا أرى كيف يمكن للجسيم الذي يعطي كتلة للأجسام أن يكون له كتلة في حد ذاته.

    يفهم؟

  16. إلى إسرائيل
    لم أفهم، وسأطرح السؤال الذي بالتأكيد هؤلاء هم أذكى مني بالفعل: إذا كان بوزون هيغز يعطي الكتلة لشخص ليس لديه كتلة فمن الذي يعطي الكتلة لبوزون هيغز؟؟ بوزون هيغز نفسه؟... فإذا كان يستطيع فلماذا لا يستطيع الجميع أن يهبوا أنفسهم للقداس؟؟ . وقد نصادف هنا مفارقة معروفة وهي "مفارقة الكتاب" التي تقول: إذا كان الكتاب يخبر كل من لا يقص شعره فمن يخبر الكتاب؟؟؟
    غذاء للفكر.
    هيا اليوم هناك محاضرة مثيرة للاهتمام في الجمعية في جفعتايم: الطاقة الشمسية النووية. بحاجة للتحضير.
    مساء الخير
    سابدارمش يهودا

  17. إسرائيل ويهوذا (المملكة المتحدة) 🙂

    إسرائيل: يتم التعبير عن ميسا أيضًا في إيدوس

    يهودا: ليس لدي مشكلة مع الجاذبية التي تدفع الجسيمات. ليس من المستحيل أن ينفذوا وينفذوا كل ما تفرضه عليهم. لديهم كتلة، ولديهم قوة دافعة، وطاقة حركية، وكل ما هو ضروري لإنتاج الفيزياء. إنهم ليسوا دقيقين بما يكفي لذوقي. أبحث عن كائن خالٍ من كل تلك الصفات الجسدية الأساسية. لأنه بخلاف ذلك لم نتقدم على الإطلاق.
    أزروم معك: دعونا نبني نموذجًا يعتمد على جسيم ليس له خصائص فيزيائية، والذي من خلال سلوكه - كفرد أو كمجموعة - يتم إنشاء جزيئات دفع الجاذبية.

  18. نعم، تتجلى الكتلة في خاصيتين:

    1. مقاومة التسارع (القصور الذاتي).

    2. الجاذبية.

    في جميع القياسات التي أجريت حتى الآن، لا يوجد فرق قابل للقياس بين هاتين الخاصيتين للكتلة، لذلك يمكننا رؤيتهما كخاصية واحدة.

    في دفع الجاذبية، بدون الجسيمات لا توجد كتلة جاذبية. يمكن أن تكون المسافة بين الأرض والشمس مترًا واحدًا دون أي تجاذب بينهما. ولذلك، مما قيل في القسم السابق، لا توجد كتلة قصورية بدون الجسيمات أيضًا.

    وبالتالي، إذا كانت الأجسام ليس لها كتلة بدون الجسيمات، فكيف يمكن للجسيمات نفسها أن تكون لها كتلة؟ ما هي الجزيئات الأصغر التي تمنحها صمامها؟ بوزونات هيغز الخفيفة؟

    ?

    ??

    ؟؟!؟

  19. لا أفهم ما هو الجسيم بدون كتلة، بقدر ما أفهم مفهوم الجسيم إذا وفقط إذا كان له كتلة. صحيح، أعلم أن الفوتون على سبيل المثال ليس له كتلة ساكنة وبعض التعريفات الغريبة الأخرى، لكن لا داعي للاعتذار لأن جسيماتي لها كتلة. "وإذا حددت أن لديه زخمًا، فهل هذا سيرضيك؟ وماذا عن الطاقة الحركية، هل يُسمح بالحصول عليها؟ إن جسيماتي لديها كل ما يلزم لدفع الجاذبية، وإذا كان هناك بوزون هيغز، فهناك مشكلة في ذلك، ثم Zebsho (إنها مشكلته)
    يرجى الرد بلطف
    يوم جيد
    يهودا

  20. نعم، إذا كانت جسيمات الجاذبية الدافعة لها كتلة وهي أيضًا مصدر الكتلة لجميع المواد الباريونية، فما الذي يعطي كتلة لجسيمات الجاذبية الدافعة؟ جزيئات أصغر؟ والتي بدورها تحصل على الصمام من جزيئات أصغر؟ بدور... بارتنكونكونز... بارتنكونونز... بدورنا..

  21. يهودا،
    ليس لدي كتلة. إذا عرفنا الكتلة عن طريق الكتلة، فإننا لم نكتسب أي شيء.
    الكتلة هي تعبير عن ظاهرة ناجمة عن سلوك مجموعة من جسيمات المادة المظلمة.

  22. يوفال المثير للاهتمام، أيضًا في دفع الجاذبية يدعي الكثيرون أن جزيئات دفع الجاذبية تشكل المادة الباريونية وعندك هي نفسها إلا أنها مظلمة ربما في النهاية هي نفس السيدة في تغيير المجد؟؟، غذاء للفكر .
    يجب أن يكون التركيز على ما تقوله: "لتحديد الخصائص المكانية والرياضية لجسيم المادة المظلمة أولاً." "انتهى الاقتباس. أو بمعنى آخر.... تحديد جسيم الجاذبية الدفع. بالنسبة لي كتلته 6.3 ضرب عشرة أس سالب 37 جرام فماذا عنك؟؟
    مساء الخير
    يهودا

  23. شكرا لك يا أبي
    المبدأ الكامن وراء النموذج الذي توصلت إليه قبل 40 عامًا والذي أعزوه اليوم إلى المادة المظلمة يعتمد على مبدأ البساطة.
    تمثل الفيزياء اليوم "حديقة حيوانات" كبيرة من الجسيمات والقوى التي تبدو من ناحية مختلفة عن بعضها البعض ولكنها من ناحية أخرى "تتحدث" مع بعضها البعض. ويشترك معظمها في خصائص معينة، أولها الخاصية الجماعية. أحد الأقسام التي أقوم بها فيما يتعلق بهذه الخاصية، الكتلة، هي تلك التي تم قياسها وتلك التي لم يتم قياسها بها أو تم قياسها بشكل غير مباشر فقط.
    يسكن الكون جسيمات المادة المظلمة ذات الكثافة التي تختلف من مكان إلى آخر. وبكثافة عالية، فإنها تشكل هياكل نعرفها باسم المادة الباريونية. وبكثافة منخفضة فإنها تخلق ما نسميه الفضاء الفارغ. في المادة الباريونية يمكننا قياس الكتلة. انتشار الموجات الكهرومغناطيسية ممكن في الفضاء الفارغ.
    للإجابة على سؤال كيف ترتب جسيمات المادة المظلمة نفسها لتشكل هياكل باريونية من جهة ومساحة فارغة من جهة أخرى، وكيف يتم بناء قوانين الفيزياء بشكل عام، يجب أولاً معرفة الخصائص المكانية والرياضية لجسيم المادة المظلمة. مُعرف. لكن يجب الحذر من إعطاءه أي خصائص فيزيائية معروفة، إذ من المفترض أن يبني الفيزياء المعروفة ولا يبنى منها.

  24. شكرا ارييل. لقد قمت بتغيير الموقع كما اقترحت وأنا أنتظر بفارغ الصبر لأرى ما سيحدث.

    شكرا لك أيضا، يهودا. دون الخوض في تفاصيل النموذج الخاص بك، سأقول فقط أنني لا أحب اختلافات الضغط التي تتحدث عنها. وذلك لسبب بسيط وهو أن الضغط ظاهرة فيزيائية تُعرّف على أنها دالة لكمية فيزيائية أساسية (القوة)، ويتم تعريفها أيضًا على أنها دالة لكمية فيزيائية أساسية (الكتلة)؛ وبهذه الطريقة يتم الحصول على تعريف دائري لا يقودنا إلى أي مكان.

  25. رافائيل
    سؤال جيد! وعندما ننظر إلى مجرة ​​أخرى، مثل مجرة ​​المرأة المسلسلة، نجد أن حجم المجرة صغير مقارنة بالمسافة. لذلك، يمكن الافتراض أن كل الضوء القادم من المجرة يصل في نفس الوقت.

    بالإضافة إلى ذلك، فإن الزمن صغير بالنسبة إلى معدل تغير الموقع النسبي للنجوم. لذا، إذا نظرنا إلى مجرتنا، فإن خطأ عشرات الآلاف من السنين لا يسبب خطأ جوهريا في مواضع النجوم.

  26. الى ارييل
    ليس من الضروري أن يكون الأس للعقد، فقد تكون هناك احتمالات أخرى لا حصر لها. على سبيل المثال، قد يكون ثابت الجاذبية G مختلفًا قليلاً (احتمالات لا حصر لها)، وما إلى ذلك، عند المسافات القريبة فحصنا ووجدنا أن صيغة نيوتن ذات القوة الثانية للمسافة مناسبة لنا، ولكنها تصل إلى ألف ضوء فقط بعد مرور سنوات على ذلك، فإن أي محاولة لاستخدام الصيغة تظهر أن هناك صريرًا في الصيغة، على سبيل المثال شذوذ بايونير، وكتلة داكنة، والمزيد.
    إلى يوفال تشيكين
    قلت: "لا يوجد حتى الآن الكثير من المعرفة حول خصائص المادة المظلمة، ولكن هناك يقين حول وجودها. وذلك بناءً على ظاهرتين ملاحظتين: غبار الجاذبية، والشذوذ في حركة النجوم عند حواف المجرات. نهاية الاقتباس.
    لقد قلت أننا سنلاحظ سحابة جاذبية، وأنت مخطئ لأن ما لاحظته هو مجرد سحابة، سواء كانت جاذبية أم لا، فهذا تفسيرك فقط. وهناك أنواع أخرى كثيرة من الغيوم، مثلاً الغيوم الناتج عن العدسة أو اختلاف الضغط... أما الشذوذ في حركة النجوم عند حافة المجرة فالنقاش يدور هنا. ما هو ولماذا يحدث؟
    مساء الخير هنا في إسرائيل
    يهودا

  27. يوفال أعتقد أن تغيير الرابط سوف يحل المشكلة. قم بالتغيير إلى الصفحة الرئيسية للعالم ولن تضطر تعليقاتك إلى انتظار الإصدار (السبب هو منع النشر)

  28. لدي سؤال يدور في ذهني منذ زمن طويل. ربما الجميع يعرف الإجابة بالفعل إلا أنا. عندما ننظر إلى الكون، فإن الصورة التي نراها ليست لقطة لنقطة زمنية معينة، فكل نقطة تنتمي إلى زمن آخر. على سبيل المثال، عندما ننظر إلى مجرة ​​ما فإن هناك نجوماً هي الأقرب إلينا، وهناك تلك النجوم البعيدة عن تلك النجوم التي تقترب من عشرات السنين الضوئية أو أكثر. وكيف يمكن تحديدها بحيث تكون المجرة بالفعل على شكل حلزوني أو أي شكل آخر يظهر لنا؟

  29. يهودا، مازلت لم تشرح معنى المسافات الكبيرة... ماذا تدعي؟ أنه كلما زادت المسافة، كلما كان الأس أصغر؟ (سيتم إعطاء الأس كدالة للمسافة)
    أم تقصد أن الأس ثابت لكنه يختلف عن 2؟

  30. مسحوق الطاقة الكهروفيزيائية، فليكن...
    أحضر أيضًا هذه الاستجابة إلى الكون الموازي، حيث لا داعي لانتظار الإصدار. لن يضرك فتح حساب فيسبوك. انه مجانا.

    "مسافات كبيرة"؟ المسافة هي مسافة من أي حجم وليس هناك سبب للقول أنه على مسافات معينة ستعمل فيزياء مختلفة عن مسافات أخرى.

    لا يوجد حتى الآن الكثير من المعرفة حول خصائص المادة المظلمة، ولكن هناك يقين حول وجودها. وذلك بناءً على ظاهرتين ملاحظتين: غبار الجاذبية، والشذوذ في حركة النجوم عند حواف المجرات.

    كيف يتشكل الجسم والجاذبية من المادة المظلمة أو لماذا أدعي أن الجسم لا ينتج الجاذبية؟ حسنًا، تتجلى خاصية الكتلة في طريقتين مختلفتين ومستقلتين على الأقل، والتي تدعي النسبية العامة أنهما جانبان لنفس الخاصية - الثبات والجاذبية - كما تتنبأ بنجاح بظاهرة تسارع الجاذبية. إن وجود وضعين مختلفين لنفس السمة يتطلب البحث عن الآلية التي تسبب كلا الأمرين وكذلك عن الظاهرة الإضافية التي تم التنبؤ بها (وأكدها إدينجتون).
    ووفقا للنموذج الذي وضعه نيوتن، فإن قوة الجذب بين جسمين تتناسب طرديا مع حاصل ضرب كتلتيهما، وتتناسب عكسيا مع مربع المسافة بينهما. وفي غياب المعرفة الإضافية، من السهل القفز إلى استنتاج مفاده أن كتل الأجسام هي التي تخلق قوة الجذب بينها، وهذا بالفعل ما يفعله الفيزيائيون حتى يومنا هذا. كما أن ظاهرة الثبات من سمات الجماهير، كما هي ظاهرة التقادم. وبسبب تشابهها مع الظواهر الفيزيائية الأخرى، فقد تم الافتراض بأن الجاذبية هي ظاهرة مزدوجة - موجة/جسيم - يتم حملها عبر وسط غير معروف عن طريق جسيمات تسمى "الجرافيتونات"، ولكن لم يتم توظيف تلك الجسيمات، في الوقت الحالي. ، لتفسير ظاهرة الثبات والاضمحلال. وحتى اليوم، لم يتم اكتشاف الجرافيتونات بعد، كما لم يتم اكتشاف الآلية التي تنتج الظواهر المختلفة المنسوبة إلى الكتلة. إن اكتشاف المادة المظلمة يعطي الأمل في أن الاكتشاف الذي طال انتظاره سيأتي قريبًا.

    لا يقتصر الأمر على لعب البوكر أيام الثلاثاء، ولا يعيش يهودا فقط في شارع إيلي كوهين في هرتسليا.

  31. للسيد مسحوق الطاقة
    سأحاول أن أشرح لك المقصود بالمسافات الكبيرة
    وفي صيغة نيوتن للجاذبية يوجد عامل R الذي يعبر عن المسافة بين الجسمين المنجذبين لجاذبية الأرض، والمسافات الصغيرة هي النظام الشمسي الداخلي تصل إلى 60 وحدة فلكية، أي حوالي عشرة مليارات كيلومتر (حوالي ألف سنة ضوئية).
    وتحسب معادلة نيوتن للجاذبية بيانات هذه المسافة مطابقة تقريبًا للواقع. صغيرة حتى لو كانت نجمتين في أندروميدا.
    لكن لنفترض أن R تساوي 60,000 وحدة فلكية - حوالي 10,000 مليار كيلومتر أي حوالي سنة ضوئية واحدة، فهل يمكننا حتى الآن التأكد من أن صيغة نيوتن ستعطي تنبؤات صحيحة؟. وما زال يعتبر مسافة
    وماذا عن R، وهي المسافة من مركز مجرتنا إلى مركز مجرة ​​المرأة المسلسلة، حوالي مليوني سنة ضوئية، أي 20 مليون مليون مليون كيلومتر، إنها بالفعل مسافة طويلة. وحتى ذلك الحين، هل سنتمكن من ذلك؟ هل تأكدت من صحة صيغة نيوتن لهذه المسافة؟ يعني حتى حينها هل ستظهر لنا قوة الجاذبية بالضبط بين مجرة ​​درب التبانة والمرأة المسلسلة؟ وماذا عن مسافات مليارات السنين الضوئية ؟؟؟؟
    هل حصلت عليه يا سيد إي باودر؟

    يوم جيد
    سابدارمش يهودا

  32. إسرائيل، يوفال، يودا
    شكرا. لكني مازلت لم أفهم معنى مقولة: "على مسافات بعيدة"؟

    اركمان
    تسريحة شعر رائعة 🙂

    إذن أنت تدعي - مثل النموذج القياسي - أن الجاذبية قوة منفصلة. يقف بمفرده، كالقوة الضعيفة مثلاً. هل فهمتك صحيح؟
    ثانيًا، كيف يتشكل الجسم والجاذبية من المادة المظلمة؟ دقة الجزيئات؟ 🙂 سأكون ممتنا إذا أمكنك التوضيح.
    بعد كل شيء، لا توجد حتى الآن معرفة حول خصائص المادة المظلمة، والأكثر من ذلك، حتى اليقين بشأن وجودها لا يزال محل شك كبير. كل ما هو معروف أن هناك تلوثا وأنه - على الأرجح - بسبب مادة أخرى. مظلم.

    شيء آخر: لماذا تزعمون أن الجسم لا يسبب الجاذبية؟ ألا تخلق كتلة الجسم قوة ما تجذب جسمًا أقل كتلة؟
    ربما أنا فقط، لكنني اعتقدت دائمًا أن الأجسام تخلق قوة تسمى الجاذبية، لكننا ما زلنا لا نعرف ما هي الجسيمات التي تشكل هذه القوة.
    حسنًا، حسنًا... الثلاثاء. لعبة البوكر.
    سأذهب إلى حديقتي لأقطف بعض الطماطم الداكنة التي يبلغ عمرها 80 عامًا وأرسلها إلى شارع إيلي كوهين في هرتسليا دون الكشف عن هويتي، على أمل أن أستعيد 80 جرامًا من البهجة التركية.
    بالمناسبة، يوفال، سأكون ممتنًا إذا قمت بالرد هنا لأنني لست مشتركًا في فيسبوك أو تويتر وكل ذلك.

  33. ألبينتيزو,
    بعد محادثتنا، تأتي الاقتباسات التالية من مدونة كارول لتوضيح سبب تساؤلي عن معنى *المتفرج* (وللآخرين الذين قد يكونون مهتمين)
    إنها نتيجة لقياس فعال للحالة الكمومية لمجال التضخم عندما يعاد تسخين الكون في نهاية التضخم

    لكن عندما لا ننظر، فإن النظام في حالته الأرضية (مثل الإلكترون في مداره الأقل طاقة حول نواة الذرة) لا يتقلب على الإطلاق؛ إنه مجرد الجلوس هناك

    لكن لا توجد أجهزة كشف للجسيمات، لذا في الواقع فإن الحقول الكمومية تجلس هناك بهدوء في حالة ثابتة دون وجود عدد محدد من الجسيمات. ولذلك، فإن هذه الأنواع من التقلبات لا "تحدث بالفعل".

    ثم قرأت هذا:
    توفر الحالات المجهرية للبلازما بيئة تتشابك مع تقلبات التضخم واسعة النطاق، مما يؤدي إلى قياسها بشكل فعال وانهيار الدالة الموجية

    هل هذا يعني أن كل "عنصر" كلاسيكي هو في الواقع مراقب؟ هل يمكنني أن أستنتج أن أي شيء غير كلاسيكي لا يمكن أن يكون راصدًا يؤدي إلى انهيار الدالة الموجية؟

    سأكون ممتنًا إذا قمت بإعادة النظر في فكرة الكون من لا شيء، لأنه إذا فهمت بشكل صحيح، فإن كارول يتحدث بالفعل عن الاضطرابات، لكنه يدعي أنها حدثت في نهاية مرحلة التضخم (وإذا فهمت صحيح، هذا هو الوضع الذي توجد فيه بالفعل هياكل كلاسيكية تعمل كمراقبين) ولكن في عالم متضخم لا يحتوي إلا على طاقة فراغية ولا كشافات، ولا تقلبات كمية على الإطلاق (هذا ما كتبه. أضفت موضوع الكشافات ). لماذا سيكون الأمر مختلفا في حالة الكون البارد (ضرب الموت)
    أنا أفهم أنه يعتمد على أكوان متعددة ولكن في ظل هذا الافتراض، إذا كان هو وزملاؤه على حق، فهل لا تزال هناك مشكلة هنا في فكرة الكون من لا شيء؟

  34. يهودا،
    يتشكل الإعصار الوحشي حيث تتركز معظم الرياح وحيث تتواجد معظم الكتلة. في مجموعة بولت، الجاذبية ليست حيث توجد معظم الجسيمات التي نعرفها. كيف تفسر ذلك؟
    لماذا تعارض المادة المظلمة (التي لا تعرف شيئًا عنها على الإطلاق) من خلال تقديم مادة مظلمة أخرى مثيرة للإشكالية ومن السهل دحضها بتجربة بسيطة كما اقترح ألبنتزن؟ كيف لا يمكنك رؤيته؟

  35. شلومي نركيس هو اسم سويدي. بالنسبة إلى تومي، اعتقدت أن اسم يوفال تشيكين يضيء ضوءًا أحمر. سأكشف أيضًا سر الحجب الانتقائي. وعلى كل حال فإنني أرد أيضا في المقالة المقابلة على الفيسبوك وهي الرابط لها
    https://www.facebook.com/hayadan/posts/1026656190686882

    شموليك،
    إن جزيئات المادة المظلمة والمسافات بينها هي التي تشكل المادة من جهة، وتشكل الوسط الذي تنتشر فيه الموجات الكهرومغناطيسية من جهة أخرى. النسبة الكمية بين الجزيئات والمساحات (الكثافة) هي التي تحدد حجم الهياكل المادية وسرعة انتقال الموجات الكهرومغناطيسية. يتكون الجسم الضخم من كثافة عالية من جزيئات المادة المظلمة. وينتج عن سلسلة التركيزات أيضًا كثافة عالية في محيط الجسم الضخم، وهذا ما يبطئ حركة الموجات الكهرومغناطيسية وينتج عنه انحناء الضوء.

    لقد أصبحت الخيمياء قديمة الطراز بالفعل، ونموذج المادة المظلمة الذي أحضره متوافق مع القانون الثاني للديناميكا الحرارية.
    في النهاية لن يكون الأمر مثيرًا للاهتمام.

  36. يهودا

    تكتب "الجزيئات نفسها تتحرك حول الشمس مثل أي مادة أخرى".

    جميل، ولكن في أي اتجاه؟ وبأي سرعة؟

    فهل من الممكن أن يكون لكل جسم يدور حول الشمس جزيئاته الخاصة المعدلة؟ وماذا عن القمر الذي يدور حول الأرض؟ ومركبة أبولو الفضائية التي تدور حول القمر؟ ودافني الذي يدير أبولو ويدير رأسه؟..

    لكن لا تيأس يا أخي. قضاء وقت ممتع مع الأبناء والأحفاد.

    هناك حل أنيق لمشكلة الاحتكاك، لكن هذا لا يعني أن دفع الجاذبية هو تفسير حقيقي.

  37. المعجزات
    هل تريد تفسيرا غير الجاذبية؟،،،، من فضلك
    تحرك إعصاران عبر المحيط باتجاه بعضهما البعض واصطدما ونشأ إعصار أكبر - وهي حالة حقيقية أتذكر معلومات شخصية. الجاذبية لم تنجح هنا الضغط التفاضلي؟ نعم!
    ويبدو لي أن هذا ممكن أيضًا بالنسبة للمجرات،
    لذلك هذا أنا أقفز إلى الأحفاد
    باي
    يهودا

  38. يهودا
    ونحن نعلم من خلال الملاحظات أن المجرات تتصادم، ونرى أيضًا في الصور الفوتوغرافية تأثير الجاذبية على شكل المجرات.

    أحب أن أسمع تفسيرا آخر لهذه الصور.

  39. شموليك وأرييل
    قد تكون على حق والمادة المظلمة موجودة ومعها طارت الطاقة إليها وشرحت أيضًا مسألة سرعة الضوء بعشرة أضعاف لذلك دعونا فقط نقول وداعًا كأصدقاء لقد تعبت لمدة أسبوع تقريبًا من مجرد الرد والوقوف تحت النار والظلام وعلي أن أنهي عملًا لعميل ينتظرها غدًا وقد وصل أحفادي الجميلون أيضًا. لذلك كان لطيفًا وممتعًا
    كل التوفيق والى اللقاء، وأرجوك لا تغضب.
    يهودا

  40. ارييل،

    أنت تدعي أنك لست فيزيائيًا، لكن ربما كان ينبغي أن تكون كذلك. باختصار، لقد قمت بتسجيل ملاحظة حول خطأ يهودا (حسنًا، أحد أخطاء يهودا الكثيرة جدًا جدًا) وأيضًا حول كيفية قياسه وإظهاره في تجربة أنه مخطئ (إذا كان سيستمع إليك فهذا شيء آخر السؤال، وتخميني هو بالطبع - لا).

    وبما أنني أكرر فقط ما قلته بالفعل، فسوف ألخصه في سطرين - ما يصفه يهودا هو توسع المادة في الفضاء اللانهائي، وليس توسع الفضاء. هذه أشياء مختلفة تمامًا. طريقة التمييز بينهما هي التحقق مما إذا كنا نشعر بالتسارع. يعرف الجسم المتسارع أنه يتسارع (وهي كمية قابلة للقياس)، ونحن نعلم بحكم التعريف أننا *لا* نتسارع خارجًا من مركز ما في الكون (في وصف يهودا يجب أن يكون هناك مركز للكتلة الغازية، أي: لتوزيع المادة في الكون، ونحن نعلم أن الأمر ليس كذلك)، بل أن الكون يتوسع في جميع أنحاء النقطة بشكل متناحٍ وأن هذا التوسع يصبح أسرع. وهذا ليس تسارعًا بمعنى الجسم المتسارع (الذي تؤثر عليه قوة)، ولكنه فضاء تطول فيه جميع المسافات بمعدل متزايد.

    لكن على الرغم من تميز آرييل الجميل، إلا أن هناك شيء أكثر أساسية يمكن قياسه في معمل مدرسة ثانوية، وهذا ما كتبته في التعليق السابق. كل ما يجب فعله لدحض التوراة الميكانيكية (وبالفعل هذه هي الطريقة الأولى لدحضها، على الرغم من وجود مجموعة كبيرة من الدحضات) هو رمي كرة في غرفة لا توجد بها مقاومة للهواء والتحقق مما إذا كان الاحتكاك غامضًا خلقت من العدم.

  41. يهودا،
    لا يهمني ما تستخدمه، إلا أن النظرية النسبية تتنبأ بنجاح بأشياء كثيرة بما في ذلك منع المادة ذات الكتلة من تجاوز سرعة الضوء. لذلك، أنت لا ترى مشكلة اقتراحك؟

    لم تجب كيف تفسر فكرتك الكتلة البارزة؟؟؟

    لم تجب كيف لا ترى التناقض القاطع في الترويج للمادة المظلمة بينما تصرخ من هنا حتى إعلان جديد بعدم وجود المادة المظلمة.
    بحكم التعريف، أنت تتحدث عن جسيمات لم نكتشفها بعد. إنه h u m r a p l !!!!

  42. لديك بعض الأخطاء التي ربما تكون بسبب عدم الفهم.
    1) التوسع الذي تصفه هو توسع المادة وليس الفضاء، وهو توسع الكون.
    2) النجم الساكن لن يشعر بالتسارع، سيبدو كما لو أن النجوم الأخرى تبتعد عنه والسبب مرة أخرى، تمدد الفضاء بين النجوم، أي ينشأ فراغ جديد بينهما. .
    3) تسارع الكون آخذ في الارتفاع. إنها لا تلعب اليويو. هناك ما يسمى بثابت هابل ويقول إنه مقابل كل مليون بيرسيك (بيرسيك = 3.26 سنة ضوئية) ستظهر سرعة الجسم بالنسبة إلينا أكبر بمقدار 69.32 كيلومترًا أخرى في الثانية.
    أي أنه كلما زادت المسافة، زادت السرعة.
    4) في النهاية سيصل توسعك إلى حالة من التوازن مع الجاذبية (سواء كنت على حق بشأن أس المسافة أم لا) وسيتم إنشاء كون ثابت.

  43. ارييل
    إن اختلاف الضغط لدي يفسر بشكل جيد التسارع في توسع الكون دون الحاجة إلى الطاقة المظلمة. بهذه البساطة لدرجة أنك تريد البكاء!
    لذا يرجى شحذ حواسك لأنني لن أشرح مرتين. طيب لنجري تجربة:-
    في حاوية مفرغة ضخمة ذات قطر لا نهائي، أدخل كرة غازية قطرها 100 سنة ضوئية، تتناثر داخلها النجوم. في البداية، ستنتشر الطبقة رقم مائة في الفراغ - الطبقة الخارجية رقم 99 لن تتحرك على الإطلاق لأن المعلومات حول الفراغ لن تصل إلا بعد عام ولا يمكنها التحرك بسرعة أكبر من سرعة الضوء. بعد عام، ستبدأ أيضًا الطبقة الداخلية 99 في التحرك، وبعد عام آخر، الطبقة رقم 98، وبعد عشر سنوات، الطبقة رقم 90. حتى الآن، هل توافق على ذلك يا آرييل؟ إذا وافقت، فأنت توافق على أن النجم في انحراف داخلي لا يتحرك، ولكن بعد بضع سنوات سيبدأ في التحرك نحو الخارج بسرعة متزايدة - وهذا يعني حركة متسارعة. لا حاجة للطاقة المظلمة والجص!
    وليس هذا فحسب، بل يوضح أيضًا طبيعة التسارع في البداية، فهو سيزداد ثم يتناقص ولكن سيكون هناك دائمًا تسارع.
    لذا هنا، يقول الجميع أن هناك طاقة مظلمة وحتى الكثير، وأنا، يهودا سابدارمش، أقول أنه لا يوجد شيء.
    لذلك، في هذه الأيام التي تنخفض فيها الأسعار في جميع البورصات في العالم، لن أستثمر في الكتلة المظلمة أو الطاقة.
    ثم يحل المساء المظلم
    وداعا للجميع
    يهودا

  44. يهودا من المسلم به أنني لا أعرف ما إذا كانت الكتلة/الطاقة المظلمة فقط هي الحل الوحيد وعلى الأرجح لا ولكني لست فيزيائيًا لذا سأنتظر ذلك. إن اختلاف الضغط لديك ببساطة لا يمكن أن يفسر تسارع معدل توسع الكون لسبب بسيط:
    يعتمد ضغط الغاز على عدة عوامل: كثافة الغاز (الكتلة/الحجم)، ودرجة الحرارة (معادلة الغاز المثالي).
    עכשיו לפי הטענה שלך הפרש הלחצים מסביר את האצה בקצב התפשטות היקום אבל ככל שהיקום גדל כך הנפח גדל, היקום מתקרר, והלחץ של הגזים שלך יורד מה שאמור להראות ירידה בתאוצה של ההתפשטות מה שלא עומד עם תוצאות הניסויים שמראה שקצב ההתפשטות תלוי במרחק(אנרגיה אפלה מסבירה هذه).

  45. النقطة العائمة والنقطة العائمة
    يبدو لي أن دفع الجاذبية الخاص بـ Le Sage يختلف عن دفع الجاذبية الخاص بي. إن كوني الغازي البسيط يتحدث عن جسيم صغير حجمه أقل من عشرة أس سالب 36 جرامًا وسرعته لا تتجاوز سرعة الضوء. فيما يتعلق بسرعة الضوء الثمينة. لقد أجريت عملية حسابية نيوتنية وحصلت على سرعة الضوء بمقدار 1.3 مرة. وأنا أعلم أن سرعة الضوء هي الحد الأقصى، لذا ربما ينبغي أن تكون السرعة النسبية أقل قليلاً من سرعة الضوء، ولكن هل يُسمح لي باستخدام النظرية النسبية عندما أقوم باختبار فكرة أجنبية؟ يبدو الأمر كما لو أنني سوف نستخدم سيارتك فقط لأنهم سيعتقدون أننا أصدقاء. يمكنهم أن يجادلوا بحق بأنني لا ينبغي أن أستخدم النظرية النسبية. لذلك تركت الأمر مفتوحًا وسيقرر الجميع ما يناسبهم.
    المعجزات
    من أين لك هذا البيان؟ ما أدعيه بناءً على دفع الجاذبية الذي يتضمن أيضًا كونًا غازيًا هو أن ما يبدو أنه تصادم مؤكد مع المرأة المسلسلة لن يكون كذلك بالضرورة. السؤال هو هل الكون غازي أم لا؟ وبالمناسبة، إذا كنت قد ذكرت ذلك بالفعل، فمن الممكن أن تكون المجرة التي تحتوي على الكثير من المادة المظلمة تدور في المادة المظلمة، وسيكون لها أيضًا تأثير ماغنوس!!!. سننتظر 4 مليارات سنة، وسنرى ونقرر!
    يوم جيد
    يهودا

  46. يهودا،

    لقد عرضت لكم بالفعل منذ سنوات قائمة من المشاكل التي ليس لها تفسير بدون المادة المظلمة. حتى أنني قدمت لك عددًا من الروابط لمقالات علمية تتعلق بالأدلة الإيجابية على وجود المادة المظلمة. لقد رفضت دائمًا قراءتها.

    هل تريد مرة أخرى؟ من فضلك. أرني كيف تشرح، بدون المادة المظلمة أو الطاقة المظلمة، طيف الطاقة لإشعاع الخلفية الكونية (وخاصة بنية صعوده وهبوطه).

  47. يهودا،

    يبدو أنك تصر على عدم فهم مسألة الاحتكاك البسيطة للغاية. قلت في أحد التعليقات الأخيرة أنه قد تكون هناك حاجة إلى تجربة. لذا، لدي تجربة لك.

    لنأخذ غرفة بها مكنسة كهربائية فوق KDA. في هذه الغرفة، أطلق كرة مدفع موازية للأرض (أسفل المحور X). وفقًا للنسبية العامة (والجاذبية النيوتونية، والتي تعد حالة خاصة لتقريب المجال الضعيف)، لن تؤثر أي قوة على المحور السيني. من ناحية أخرى، على المحور الخامس (شعاعي من مركز المركز)، ستكون هناك قوة ثابتة تقريبًا بتسارع يبلغ حوالي 10 أمتار في الثانية المربعة. أي أن هذه النظرية تتنبأ بمسار معين - مسار مكافئ مثالي.

    وتتنبأ النظرية الميكانيكية للجاذبية أيضًا بمسار معين، ولكنه مختلف. لماذا؟ بالإضافة إلى قوة الجاذبية التي يتنبأ بها (والتي في هذه الحالة هي نفس الجاذبية النيوتونية، أو في لغتك - المسافات القصيرة)، فإنه يتنبأ أيضًا بالاحتكاك في المحور السيني مقابل اتجاه الحركة. والتفسير الحرفي لذلك موجود بطريقة لطيفة وبسيطة للغاية في محاضرة فاينمان التي ربطتها بها - كرة المدفع تتحرك في اتجاه معين وفي اتجاه حركتها تصطدم بجسيمات أكثر بكثير مما في الاتجاه المعاكس (لأن يطاردهم ولا يهرب منهم كما في المطر). وبالتالي فإن فرق الضغط الذي تحبه كثيرًا سيدفعه مرة أخرى إلى البرميل.

    هذه تجربة بسيطة يمكن إجراؤها في أي مختبر. في الواقع، عندما كنت أقوم بتدريس المختبرات كطالب دراسات عليا في إحدى الجامعات في مكان ما هنا، قمت بإجراء تجربة مشابهة جدًا. الآن سأدعك تخمن - عندما تقوم بهذه التجربة على سطح الأرض، هل تجد أن كرة المدفع تتحرك بحرية على المحور السيني بدون قوى، أم أن هناك قوة احتكاك غامضة تبطئ حركتها هذا المحور رغم أنه فراغ؟

  48. الى ارييل
    إذا أظهرت لي مشكلة لا يمكن حلها إلا بمساعدة الكتلة والطاقة المظلمة، فأنا أعتذر للجميع وأدعوك لتناول وجبة ممتعة. ليس لديك أدنى فكرة عن العبء الذي سترفعه عن كتفي.
    يهودا

  49. ألبينتيزو,
    شكرا على الإجابات. سأجد محاضرة جيمس جيتس حول هذا الموضوع. لقد كانت أكثر تفصيلاً قليلاً. فيما يتعلق بلغة كارول، أحتاج إلى التفكير قليلاً فيما يزعجني (إذا فهمت بشكل صحيح، فهو يشرح لماذا لن تتشكل أدمغة بولتزمان ولكنها تسمح بتكوين الكون بأكمله؟)

  50. يهودا،
    وبعيدًا عن كل ما كتبه الآخرون، يجب أن أشير إلى أنني لم أسمع قط في حياتي عن عملية حسابية يمكن أن تنتج سرعتين مختلفتين. ما هذا، برنامج كما يحلو لك؟ لم أسمع في حياتي قط عن مثل هذا الإيمان القوي بوجود جسيمات تطير بسرعة قريبة من سرعة الضوء على كلا الجانبين، وهو ما لا نكتشفه، بينما نكره إلى ما لا نهاية ما يسميه الفيزيائيون المادة المظلمة. أنت حقًا لا تدرك أنك تستبدل المادة المظلمة التي يدعي الفيزيائيون أنها يمكن أن تفسر مجموعة الرصاصات بمادة مظلمة أخرى والتي وفقًا لفيكي (الذي هو، ما يجب فعله، أكثر موثوقية منك) يجب أن يكون له سرعة أعلى بكثير من الضوء ، في حين أن سرعة الضوء لديك قريبة من سرعة الضوء على كلا الجانبين (اعتمادًا على عدد المفاتيح الموجودة لديك) وذلك على أية حال، إذا كانت أسرع من الضوء فإنها تتعارض تمامًا مع النسبية ولا يمكنها تفسير مجموعة الرصاص لأن ما يحدث هناك هو أن معظم الجاذبية ليست حيث توجد معظم المادة المعروفة. كيف يمكن لجسيماتك تفسير ذلك؟ اختلاف الضغط لا يمكن تفسيره حقًا. فرق الضغط هو مجرد تعويذة.
    ألا تدرك أن نظرية المجال والنموذج القياسي يضعان حدًا طاقيًا للجسيمات التي تتفاعل مع المادة المعروفة ولا يمكن لأي مرشح جديد أن يقوم بالتفاعل الذي تطلب من المادة المظلمة أن تقوم به؟

  51. يهودا،
    وبعيدًا عن كل ما كتبه الآخرون، يجب أن أشير إلى أنني لم أسمع قط في حياتي عن عملية حسابية يمكن أن تنتج سرعتين مختلفتين. ما هذا، برنامج كما يحلو لك؟ لم أسمع في حياتي قط عن مثل هذا الإيمان القوي بوجود جسيمات تطير بسرعة قريبة من سرعة الضوء على كلا الجانبين، وهو ما لا نكتشفه، بينما نكره إلى ما لا نهاية ما يسميه الفيزيائيون المادة المظلمة. أنت حقًا لا تدرك أنك تستبدل المادة المظلمة التي يدعي الفيزيائيون أنها يمكن أن تفسر مجموعة الرصاصات بمادة مظلمة أخرى والتي وفقًا لفيكي (الذي هو، ما يجب فعله، أكثر موثوقية منك) يجب أن يكون له سرعة أعلى بكثير من الضوء ، في حين أن سرعتك قريبة من سرعة الضوء على كلا الجانبين (اعتمادًا على عدد المفاتيح الموجودة لديك) وأنه على أية حال، إذا كانت أسرع من الضوء، فإن زيهاس يناقض النسبية تمامًا ولا يمكنه تفسير العنقود النقطي لأن ما يحدث هناك هو أن معظم الجاذبية ليست حيث توجد معظم المادة المعروفة.
    ألا تدرك أن نظرية المجال والنموذج القياسي يضعان حدًا طاقيًا للجسيمات التي تتفاعل مع المادة المعروفة ولا يمكن لأي مرشح جديد أن يقوم بالتفاعل الذي تطلب من المادة المظلمة أن تقوم به؟

  52. للمعجزات
    سأبدأ مع أندروميدا. لا تكن على يقين من أننا سوف نصطدم بها. ليس لدي جاذبية والكون الغازي مع دوران المجرات سينتج، كما تعلمون، تأثير ماغنوس وقد تخيب المجرات آمال نيوتن. لأنه ما هي فرصة أن تلتقي كرتان مدورتان؟. ضئيلة!.
    وعلى سؤالك الأول. عندما كتب أينشتاين نظريته أضاف ثابتًا كونيًا واعتذر عنه لاحقًا، وفي النهاية كان على حق. ما يجب القيام به هو رقعة صغيرة تناسب رقعتي الصغيرة. أنظر، حتى في الختان يتم إجراء تصحيحات في بعض الأحيان.
    الحقيقة هي أنني لم أهتم بهذا "الترفيه" - كنت أقصد: "دفع الجاذبية النسبية" (جميع الحقوق محفوظة للاسم) وربما يجب عليك حقًا أن ترى ما هو جوهر التغيير الذي يجب القيام به.
    هيا، لقد ذهبت عطلة نهاية أسبوع أخرى!
    يهودا.

  53. "بعد كل شيء، النظام الشمسي يعرف دائمًا كيفية التغلب على الاحتكاك"، صحيح، فمن المحتمل أن يكون الاحتكاك صغيرًا بدرجة كافية - ثم تقبل أنه لا يوجد دفع جاذبية (لمزيد من التفاصيل: https://en.wikipedia.org/wiki/Le_Sage's_theory_of_gravitation#Later_assessments).

    على كل حال سأنتظر بفارغ الصبر الحل الأكثر دقة..

  54. يهودا
    وشيء صغير آخر. أنت تقول، كما أفهمها، أن صيغة الجاذبية خاطئة على مسافات تصل إلى عشرات أو مئات السنين الضوئية.
    فلماذا سنصطدم بأندروميدا، حتى قبل أن ينتهوا من السكك الحديدية الخفيفة في تل أبيب؟

  55. يهودا
    النسبية العامة تعبر عن الجاذبية باستخدام صيغة "بسيطة" تعطي نفس نتيجة معادلة نيوتن. أفهم أن تاثا يقترح تعديل الرياضيات بدلاً من المادة المظلمة. من الصعب بالنسبة لي أن أتقبل ذلك لأننا لم نجد علامات جسيمية لتلك المادة المظلمة في المسرعات، وأن حسابات أينشتاين خاطئة.

    لقد كان نيوتن مخطئًا، لأن افتراضاته الأساسية لم تكن دقيقة. ما هو الافتراض الأساسي الخاطئ في نظر أينشتاين؟

  56. أشكر إسرائيل على ذكر مدينة دنيدن. من الآن فصاعدا أود استخدام هذا اللقب المحبب بدلا من المصطلح الكئيب "الظلام".
    إن جزيئات الدنيدين والمسافات بينها هي المسؤولة عن الظواهر الفيزيائية المختلفة، ويتم تحديد الفروق بين الظواهر المختلفة من خلال النسبة بين الدنيدين والفراغات، أي الكثافة.
    السبب وراء عدم العثور على مدينة دنيدن في المنطقة المجاورة لنا هو على وجه التحديد أنها غير مرئية. وكما يمكن اكتشاف دنيدن (Un Sharig) لشراغا جافني عن طريق الحيل، كذلك للكشف عن دنيدن الخاصة بنا، نحتاج إلى حيل أثق في العبقرية البشرية للعثور عليها.

  57. قل يهودا، من هو العبقري الموهوب الذي صور محاضرتك على اليوتيوب؟ جودة التصوير مروعة بكل بساطة، بدءًا من الدقيقة الثالثة بالكاد يمكنك رؤية الصور الظلية، وكذلك جودة الصوت في وجهك.

    كيف يتمكن الشخص من تحقيق مثل هذه النتيجة عالية الجودة؟

    على الأقل وبخته؟

  58. إلى نقطة عائمة وإلى إسرائيل
    الاحتكاك موجود منذ تكوين النظام الشمسي وهو موجود حتى اليوم حتى دون أن يتفق مع الجاذبية الدافعة، وفي النظام الشمسي والكواكب تغلبت عليه، على الأقل بعض الكواكب. وأي شخص يتحرك في مدار مناسب من الفضاء والسرعة يستمر في الدوران حول الشمس. لقد أثرت مسألة الاتجاه بحيث لا بد من وجود متجه للقوة في اتجاه الحركة. ربما هذا صحيح، لم أتعمق فيه. ربما يكون الحل المحتمل هو قوة الطرد المركزي للمجرة بأكملها أو ربما تدور الجسيمات نفسها أيضًا حول الشمس مثل أي مادة أخرى، أي أنه لا يوجد سبب يمنع دفع الجاذبية أيضًا من التأثير على الجسيمات نفسها، وحقيقة أنها صغيرة لا ينبغي أن يكون لها تأثير، وحجم لا يهم. وربما شيء آخر؟ والأكثر من ذلك، أن النظام الشمسي يعرف دائمًا كيفية التغلب على الاحتكاك وليس هناك سبب يمنعه من القيام بذلك حتى اليوم وفقًا لقوة الجاذبية.
    بشكل عام، لاحظ أن الكواكب والأقمار، معظمها كلها، تدور في نفس الاتجاه، لذلك هناك سبب لذلك.
    لن أعطي أهمية كبيرة لمشكلة الاحتكاك، لكنها غذاء للتفكير وستشغلني.
    أرجو أن تكونوا متفائلين وأن هذا لن يسبب احتكاكاً بين المستجيبين.
    إذن، النقطة العائمة وإسرائيل، آمل أن أكون قد أجبت على الأقل جزئيًا أو مؤقتًا على مشكلة الاحتكاك.
    يوم جيد ومشرق
    يهودا

  59. إلى والدي بيليزوفسكي
    إجابتي تنتظر قرارك. ومن أجل حسن النظام، أقترح عليك تقديم قائمة بالقواعد التي من شأنها أن تسهل الأمر عليك وعلينا. قل ما تتوقعه من الرد. على سبيل المثال، ربما طوله وما شابه ذلك. ليست كل القواعد لأنني متأكد من أن لديك بعض القواعد السرية. شيء آخر، ربما يكون من المستحسن أن تختار أسماء المشاركين الذين ستثق بهم حتى "يتسخوا" مرة واحدة.
    تتم مشاركة رأيي أيضًا من قبل المعلقين الآخرين
    يوم جيد والدي
    واستمروا في العمل الجيد
    مع التقدير!
    يهودا

  60. وفيما يلي رأيي في أحدث التعليقات

    المعجزات
    لقد حصلت على النسبية ولكنها تحتاج إلى ترقية في المسافات.

    ليازي
    في نظام أندروميدا الشمسي، الجاذبية هي نفسها الموجودة في نظامنا الشمسي، لكن التجاذب بين النظامين الشمسيين يختلف عن نظامنا الشمسي. وهذا هو المقصود بمصطلح "على مسافات بعيدة". أي أنه لا يوجد اتفاق على أنه إذا كانت المسافة بين نجمين آلاف السنين الضوئية فإن الجاذبية بينهما ستكون حسب صيغة نيوتن.
    الى ارييل
    إذا كان الكون مليئا بالجزيئات التي تتحرك من مكان إلى آخر، وتصطدم ببعضها البعض، فإنها تعرف الغاز، وبالتالي فهي تتمتع بجميع خصائص الغاز مثل: الحجم، والجسيمات في المتر المكعب، وأيضا الضغط و فروق الضغط والرياح التي تعمل لملايين السنين وتحدد الحقائق في الميدان.
    شموليك
    قلت من وجهة النظر النيوتونية إنها تتحرك بسرعة 1.3 مرة سرعة الضوء، لكنني أضفت على الفور أنها من وجهة النظر النسبية تتحرك بسرعة قريبة من سرعة الضوء. افهم أن هناك مشكلة في مثل هذا التصريح، لأنني آخذ استنتاجات من نظرية علمية موجودة وأقبلها في فكرة علمية أقوم بتطويرها وليس بالضرورة أن يكون الأمر كذلك. ولهذا السبب انفصلت وقلت الخيارين.
    ارييل
    لم أغير استنتاجاتي وفقًا لألبانزو، لذا يرجى تصحيح نفسك. فكرة فرق الضغط تراودني منذ سنوات ولا علاقة لها بالبنزو.
    وأما التفسير الأنيق للكتلة المظلمة فلن يتفق عليه الجميع. على سبيل المثال، لن توافق الحمير على تناول النخالة الداكنة فقط حتى لو أخبرتهم أنها تملأ الكون بأكمله وأنها مغذية بشكل أنيق مثل النخالة البنية.
    إلى إسرائيل
    ومن المفترض أن يتم العثور على المادة المظلمة من خلال حسابات الجاذبية بشكل رئيسي عند حافة المجرة أو في السحب الغازية خارجها. بالقرب من الأرض يكون "رفيعًا".
    إلى النقطة العائمة
    هل تتحدث عن الصبر؟؟، لقد تم البحث عن المادة المظلمة لمدة ثمانين عامًا، دون نجاح كبير، ليس لدي مشكلة إذا كنت تريد منا أن ننتظر بصبر لبضع ساعات أخرى!
    يوم جيد للجميع
    سابدارمش يهودا

  61. شموليك،

    فيما يتعلق بالأمرين.

    1. لنبدأ بالفيديو. الانطباع الأول هو بالطبع أن نيل ديجراس تايسون (على الرغم من مساهمته الهائلة في العلوم الشعبية وجلب الفيزياء إلى الأشخاص الفضوليين الذين ليس لديهم بالضرورة المعرفة الرياضية اللازمة لقراءة المقالات)، فهو ببساطة يحول القصة بأكملها إلى قصة واحدة. نكتة كبيرة ووضع حاجته للترفيه بعيدًا عن اللوحة. أنا لا أهاجمه فحسب، بل إنه ذو صلة كبيرة بما سأقوله: على الرغم من أن الفيديو مدته 12 دقيقة، فإن الموضوع الذي سألتني عنه بالكاد يستغرق نصف دقيقة. المعلومات قليلة جدًا، لذا من الصعب الإجابة عليها. ولكن من حيث المبدأ - الضوضاء والرنين. لقد استمتع تايسون حقًا باللعب أمام الجماهير وإبهارهم، ولكن في النهاية رأيت أنه لم يكن أحد متحمسًا جدًا، والسبب بسيط. هناك العديد من المعادلات التي يمكن كتابتها، ومن بين العديد منها يوجد اتصالات (تحويلات) تنقل حلول إحداهما إلى الأخرى. مجرد حقيقة أن خوارزمية معينة (هناك الكثير منها) تحول المعادلة إلى شيء مرئي تعطي نتائج مماثلة في حالتين، لا يعني أن هناك علاقة سببية عميقة بين الفيزياء وراء المعادلات المعنية (لاحظ أيضًا أنه يدعي فقط أن رسومات المعادلات متشابهة، تايسون هو من حول هذا الادعاء إلى "نفس المعادلات"). ومن الممكن أن يكون للمعادلتين تماثل معين يربط بينهما، لكن هذا لا يعني أن الكون عبارة عن رمز. بالإضافة إلى ذلك، هناك فرق كبير اعتمادًا على المعادلات التي تظهرها. على سبيل المثال، إذا كانت تقدم معادلات الحركة (التي تخبرنا كيف يجب أن يتحرك الجسيم المادي) أو معادلة تصف الإنتروبيا المتشابكة في نظرية التناظر الفائق. في المعادلة الثانية هناك الكثير من العوامل البشرية - الكثير من الأشياء التي قمنا بتعريفها (في الواقع، مفهوم الإنتروبيا بأكمله مصطنع. مفيد جدًا، ولكنه مصطنع، مثل مفهوم القسمة على سبيل المثال). في هذه الحالة من الممكن أن تكون العلاقة بين المعادلات نتيجة لتعريفاتنا. أخيرًا، يجب أن نعلق على جانب تصحيح الخطأ: أفترض أن المعادلات التي يتحدث عنها هي معادلات تحسين لخوارزمية أو أخرى. يمكن أن ينشأ التشابه من حقيقة أنه تحسين، أو من اختيار الخوارزمية. في الختام، فإن التشابه بين المعادلات (أو بين عرضها البياني) ليس له أي معنى مسبق في الفيزياء. قد يكون هناك مليون سبب لذلك، ومعظمها ظرفية بحتة.

    فيما يتعلق بلغة كارول، لم أقرأ المقال، ولكن من خلال قراءة المدونة، من الواضح أن هناك بعض الأشياء هنا التي يجب الإشارة إليها. أولاً، هناك الكثير من الدلالات هنا. لاحظ أنه يقول صراحة أن نماذج الاضطرابات الصغيرة في التضخم دقيقة أيضًا وفقًا لطريقته، فهو ببساطة لا يسميها تقلبات الفراغ الكمي، ولكن فك الترابط الذي ينشأ من الأكوان المتعددة (والتوسع فيها ودرجة الحرارة) التغيير) والذي يؤدي في الواقع إلى قياس ذو طبيعة إحصائية. رياضيًا لا يوجد فرق هنا، وهذا سيكون صحيحًا أيضًا (في فهمي، هناك مكان لقراءة المقال كاملاً بالطبع) حتى لو قمنا بربط كون ضخم بكون صغير بشكل مطابق، وهو ما يتحدث عنه بنروز . لدي شعور بأن الأمر لن يهم بالنسبة لصورة لورانس كراوس أو الصور الأخرى للكون الناشئ، ولكن مرة أخرى - عليك أن تقرأ ولم يكن لدي الوقت لذلك حتى الآن.

    بالإضافة إلى ذلك، فإن الاختلافات التي يخطئ كارول في وجودها، تنبع مباشرة من افتراضه حول تطور الحالة الكمومية في الزمن، والتي تنبع من دعمه لتفسير معين لميكانيكا الكم (على وجه التحديد، حالة معينة من العوالم المتعددة). مما يعني أنه إذا كان على حق، فمن الجميل جدًا أن نجد اختلافات وصفية بين التفسيرات، لكن ليس من الواضح كيف يمكن بناء مثل هذا النموذج دون معرفة ما إذا كان التفسير صحيحًا. أعني أن هناك نوعًا من الدائرة هنا وليس من الواضح كيف تم كسرها (في الوقت الحالي، لا أدعي أنه لا يمكن كسرها).

    آسف لأنني لم أساعد أكثر، ولكن لإعطاء إجابات أفضل تحتاج إلى دراسة الموضوع (وهذا صحيح دائمًا، وأتمنى أن يكون واضحًا للجميع كما هو واضح للأغلبية الساحقة من الناس). سأحاول هذا الأسبوع الخوض على الأقل في موضوع كارول (وهو الأمر الأكثر إثارة للاهتمام بالنسبة لي) وربما أستطيع التوسع فيه. في هذه الأثناء، إذا كانت لديك أسئلة محددة، يمكنك محاولة طرحها على الرغم من أنني سأجيب على ذلك على الأرجح للإجابة، فأنا بحاجة إلى قراءة المقالات وإجراء العمليات الحسابية.

  62. في الحقيقة، في إسرائيل، تم البحث عن بوزون هيغز لمدة 40 عامًا، وما زالوا لا يضمنون بنسبة 100٪ أنهم عثروا عليه. تم البحث عن النيوترون لمدة 20 عامًا.. هل أنت متأكد من أنه "من المستحيل العثور على بعض منه"؟ قليل من الصبر 🙂

  63. يوفال

    إذا كانت المادة المظلمة في نظرك موجودة داخل النظام الشمسي وتشكل نسبة كبيرة من إجمالي المادة في الكون، فلماذا لا يمكن العثور على بعضها هنا في إسرائيل أو في الطريق إلى القمر على سبيل المثال؟

    من الصعب الجدال مع دنيدن غير المرئي الذي يفوق وزن المرآة بـ 70 كيلو.

  64. يهودا، في البداية زعمت أن المشكلة التي لدينا هي أن الجاذبية على مسافات كبيرة كما نصفها (مربع المسافة) غير صحيحة. بعد أن أظهر لك ألبينزو أن تغيير الأس من شأنه أن يسبب المزيد من المشاكل، أضفت أيضًا فرق الضغط ودفع الجاذبية إلى المعادلة (على الرغم من أن هذا يتعارض مع تحفظك على الكتلة المظلمة) وما زلت تواجه مشاكل (مرة أخرى، شكرًا جزيلاً لألبينزو ). ذكرني كيف يكون هذا أكثر أناقة ومنطقية من إضافة كتلة داكنة إلى حساباتنا؟

  65. بعد أن تحدثنا عن الجسيمات المظلمة التي هي أسرع 1.3 مرة من الضوء (!) وهي ليست المادة المظلمة (!)، علمنا أن المادة المظلمة بجانب جسم ضخم هي المسؤولة عن انحناء أشعة الضوء وليس فالجسم الهائل والمسافات بين الجزيئات هي الوسط الذي يحمل الموجات الكهرومغناطيسية، وسنتحدث عن الخيمياء واستخداماتها المختلفة في التغلب على القانون الثاني للديناميكا الحرارية.
    سيكون من المثير للاهتمام

  66. ليس لدينا أي سبب لنفترض أن الفيزياء الأخرى في المجرات الأخرى اعتباطية، ولكن بما أن المجرات تختلف عن بعضها البعض في الحجم والكتلة، فيُسمح لنا أن نفترض أن كل مجرة ​​لديها فيزياء ثابتة فريدة خاصة بها.

  67. يازي، لفهمك أن الجاذبية هي نتاج جسم/عظم:
    فوجود الجسم هو نتاج الكثافة المحلية للمادة المظلمة، كما هو الحال مع وجود الجاذبية. دعونا نفهم، ليس وجود الجسم هو الذي يخلق الجاذبية، ولكن وجود الجاذبية ووجود الجسم كلاهما مخلوقان في نفس الوقت من عامل ثالث - يعرف أيضًا بالمادة المظلمة.

  68. تفضل
    تسحب الأرض القمر بنفس القوة التي يجذب بها القمر الأرض. كلاهما يدوران حول بعضهما البعض، ويكون محور الدوران عند مركز الثقل المشترك. والأمر أكثر تعقيدا من ذلك، لأن الحركة بيضاوية.

    يتفق الجميع على أن قانون الجاذبية في كل مكان، وعلى مسافات قصيرة، هو قانون نيوتن، أي أن القوة تعتمد على الكتل ومربع المسافة.

    والسؤال هو ما إذا كان هذا القانون صالحًا أيضًا للمسافات الكبيرة. السؤال مطروح - لأننا نرى ظواهر غريبة على مسافات بعيدة. والتفسير المقبول هو أن هناك مادة إضافية لا نراها مباشرة، وهذه المادة لها كتلة. وهناك ظواهر أخرى تفسرها هذه المادة ونعرف الكثير من خواصها.

    تفسير آخر هو ما يقوله يهودا - قانون نيوتن للجاذبية غير صالح على مسافات كبيرة.

    أنا لست فيزيائيًا، لكن عندما يقول 99.9% من الفيزيائيين شيئًا ما، أراهن أنهم على حق. ومن الصحيح بشكل خاص أن التفسير المنافس يبدو غير منطقي بالنسبة لي.

  69. تفضل
    "ولأنه أكثر ضخامة، فإنه يجذب القمر وليس العكس."

    والعكس أيضًا، ولكن فقط بسدس القوة التي تجذب الأرض القمر نحوها.

    "هل يعني ذلك أن الجاذبية في أندروميدا تعمل بشكل مختلف أم أنها غير موجودة على الإطلاق أم أن هناك شيئًا آخر؟"

    لا أعتقد أن هذا ما يدعيه يهودا. لكن إذا تعاملنا مع الجاذبية كقوة دافعة - وهو ما تدعيه نظرية الجاذبية الدافعة - فيمكنها منطقيا تسليط الضوء على شذوذ دوران النجوم عند حواف المجرات، وهو ما يشكل المحفز لنظرية المادة المظلمة، على الرغم من أن وهناك أسباب أخرى تبرر وجودها.

    المشكلة الرئيسية التي وافق عليها يودا هي أنه لا يزال من المستحيل الذهاب إلى السوبر ماركت وشراء 2 كيلو من المادة المظلمة وسلة من الجرافيتون.

  70. يهوذا، إسرائيل، يوفال
    حسب فهمي، الجاذبية هي نتاج الجسم/العظم. كلما زادت كتلة الجسم، كلما أنتج الجسم قوة تجذب إليه أجسامًا أخرى (ذات كتلة أضعف منه).
    توجد بين الأرض والقمر قوة جاذبية تتكون من كتلة الأرض والقمر. وبما أنه أكثر ضخامة فهو يجذب القمر وليس العكس. وبطبيعة الحال، تشارك أيضًا أجرام سماوية أخرى في هذه العملية، مثل الشمس على سبيل المثال.
    أي أنه لا يهم إذا كان هنا أو في مجرة ​​المرأة المسلسلة، بين الأجسام هناك قوة جذب تنشأ من الأجسام نفسها وبسببها.
    في فهمي، حتى الجسيم الأولي لديه قوة تجذب جسيمًا آخر إليه.
    ما لا أستطيع أن أفهمه هو ما تقصده عندما تقول "على مسافات بعيدة".
    هل المقصود أن يكون وسيطاً بين جسد وجسد آخر؟ هل يعني ذلك أن الجاذبية في أندروميدا تعمل بشكل مختلف أم أنها غير موجودة على الإطلاق أو شيء آخر؟

  71. يهودا
    لا يعمل بالنسبة لي. في النسبية العامة، كما أفهمها، فإن قوة الجاذبية لا تنتهي عند أي مسافة. اترك في الوقت الحالي الملاحظات التي تظهر أن هناك ظواهر أخرى - فالنظرية النسبية تقول شيئًا محددًا للغاية.

    إما أن تقبل النظرية النسبية أو لا تقبلها.

  72. ماذا يحدث لك 🙂
    في الحركة الدائرية توجد قوة طرد مركزية. في حالتنا الجاذبية هي قوة الجذب المركزي.
    قوة الطرد المركزي هي قوة خيالية، هل نسيت؟

  73. إسرائيل، على تساؤلك "نحن نحاول فقط أن نفهم كيف أن الأرض تسحب القمر إذا لم يكن هناك شيء بينهما، ودفع الجاذبية يعطي وصفًا جميلاً وواضحًا. لكن هناك مشكلة الاحتكاك الأساسية، ومجموعة من المشاكل الثانوية"، أمران:
    أ) حقيقة أننا لم نكتشف بعد وجود شيء ما في المنتصف لا تعني عدم وجوده بالفعل. إن وجود المادة المظلمة في الفضاء داخل المجرة هو بالفعل حقيقة مثبتة بالملاحظة، ولا يوجد سبب للافتراض بأنها غير موجودة أيضًا داخل النظام الشمسي وفي جميع مناطق الكون بشكل عام.
    ب) إن ظاهرة القوى الجاذبة والتنافرية، وكذلك ظاهرة الاحتكاك، محددة رياضيا في الفيزياء المعروفة. إن محاولة تعريف أنفسهم باستخدامها أمر دائري وغير ضروري.

  74. يهودا، كتبت: "هناك ثلاث قوى تعمل هنا، الجاذبية الاحتكاكية والجاذبية الطاردة المركزية، وستكون الحركة هي الناقل المتوازن لكل هذه القوى. لا أرى مشكلة." إذا لم يشير المتجه المتوازن إلى اتجاه مركز الدائرة، فمن المؤكد أن هناك مشكلة، لأنه لن يتم الحفاظ على المسار الدائري. ولديك قوة ضارة تسمى الاحتكاك وهي متعامدة.

  75. إسرائيل سبقتني في مسألة الاحتكاك أما يهودا فلابد من صقل شيء..

    لقد كتبت "قوة الطرد المركزي للكوكب، على سبيل المثال، ستكون مساوية للجاذبية الناتجة مطروحًا منها الاحتكاك". هل نسيت ما هو الاحتكاك؟ قوة الطرد المركزي تكون عمودية على اتجاه الحركة، والاحتكاك يكون مع اتجاه الحركة! لا يمكنك طرح الاحتكاك من الجاذبية إذا كانا متعامدين على بعضهما البعض.

    إسرائيل على حق – للتغلب على الاحتكاك يجب أن تكون هناك رياح إضافية مع اتجاه الحركة، وكما كتب، فإن هذا يخلق وضعا سخيفا.

  76. إلى إسرائيل
    أنا لا أتفق معك. السفن الشراعية قادرة أيضًا على التحرك عكس اتجاه الريح.
    هناك ثلاث قوى تؤثر هنا، وهي الجاذبية الاحتكاكية والطرد المركزي، وستكون الحركة هي الناقل المتوازن لكل هذه القوى. لا أرى مشكلة.
    يبدو أنه يجب علينا إجراء تجربة
    حسنًا، حان وقت النوم
    مساء الخير
    يهودا

  77. يهودا

    تحتاج السفينة التي تبحر في حركة دائرية إلى مصدر للطاقة للتغلب على الاحتكاك مع الماء.

    إذا كانت "الريح" بالنسبة لك هي مصدر الطاقة، فيجب أن تكون بالضبط في اتجاه حركة السفينة، وهو أمر مستحيل لأن كل سفينة أخرى، بما في ذلك تلك التي تبحر في الاتجاه المعاكس، ستحتاج إلى الرياح في اتجاهها.

    شيء آخر هو أن الاحتكاك يولد الحرارة، حتى في مثال سفينتك الشراعية، والحرارة هي طاقة يجب أن تأتي من مكان ما.

    في مثال السفينة، الريح هي مصدر الطاقة. وهذا لا يمكن أن يوجد في حالة جسيمات لازاج، لأنه كما ذكرت فإن الحركة الدائرية في الاتجاه المعاكس ستستهلك أيضًا الطاقة من تلك الجسيمات.

    يوفال

    من أين أتت الفيزياء إلى العالم - سؤال محترم للبشر البسطاء مثلنا.

    نحن نحاول فقط أن نفهم كيف أن الأرض تسحب القمر إذا لم يكن هناك شيء بينهما، ويعطي دفع الجاذبية وصفًا لطيفًا وواضحًا. ولكن هناك مشكلة الاحتكاك الرئيسية، ومجموعة من المشاكل الثانوية.

  78. إسرائيل شابيرا
    ووفقاً لهذا المنطق فإن السفن الشراعية لن تستطيع الإبحار في البحر لأنها تحتك بالماء والهواء؟؟؟. السبب وراء عدم توقفهم عن الإبحار هو أن الرياح تهب باستمرار وتتغلب على الاحتكاك، وبالتالي حتى في حالة دفع الجاذبية، طالما استمرت الجزيئات في الوصول إلى النظام، فإنها ستستمر في خلق الجاذبية التي ستنجح في التغلب على احتكاك. آمل ألا أضطر إلى شرح ذلك مرة أخرى.
    الى ارييل
    على سؤالك الأول
    ليست هناك طرق سهلة ولكنها ممكنة لإثبات الأفكار التي تكمن في دفع الجاذبية أو على الأقل حول عدم صحة مربع مسافة الجاذبية على مسافات كبيرة. على سبيل المثال، من المفترض أن يدور النظام المزدوج double epsilon lira بشكل أبطأ مما هو عليه وفقًا لنيوتن وبعض الخيارات الأخرى
    وحول السؤال الثاني
    إن فرق الضغط الموجود منطقيًا في الكون المليء بالجسيمات المتحركة يمكن أن يخلق حركة دون الحاجة إلى جذب الكتلة. يمكن لفرق الضغط أن يفسر التوسع المتسارع للكون دون الحاجة إلى كتلة مظلمة، ولكن هذا لمحاضرة كاملة.
    مساء الخير
    يهودا

  79. الحديث عن "احتكاك" أو "امتصاص" "جزيئات الغاز" هو محاولة لتعريف الفيزياء باستخدام الفيزياء. هذا تعريف دائري سيترك دائمًا طعم "المزيد" ولن يحل مسألة كيفية ظهور الفيزياء إلى العالم.

    في ذلك الوقت تشاجرت مع يوآف ليتبيك الذي قدم نموذجًا مصورًا جميلًا يدعي أنه يفسر الظواهر الفيزيائية الأساسية من خلال ظواهر فيزيائية معقدة. انتهى الجدال بيننا بالانفصال، وهو ما يؤسفني. لكن النقطة التي أصر عليها هي أن قوانين الفيزياء المعروفة اليوم هي نتيجة لقوانين أساسية في الفيزياء لا تزال مجهولة بالنسبة لنا.
    أنا على استعداد للموافقة على شيء واحد، وهو أنه حتى القوانين الأساسية للفيزياء التي لم تُعرف بعد يجب أن تستجيب لأي صيغ رياضية للقياس الكمي.

  80. النسبية العامة هي نموذج يحدد كمية الظواهر الفيزيائية ولكنه لا يفسرها.
    يتم تفسير "انحناء" الضوء أثناء مروره بالقرب من جسم ضخم من خلال سلسلة كثيفة من المادة المظلمة. يتم التعبير عن كثافة كبيرة في الأجسام المعروفة في الفيزياء المعاصرة؛ يتم التعبير عن الكثافة المنخفضة فيما تعرفه الفيزياء اليوم بـ "الفضاء الفارغ". تقوم جزيئات المادة المظلمة ببناء الأجسام المادية؛ الفراغات بين الجزيئات هي الوسط الذي يحمل الموجات الكهرومغناطيسية.

  81. يهودا

    الجسم الذي يتحرك في الهواء، بما في ذلك الحركة الدائرية، سوف يتباطأ ويتوقف بعد فترة طويلة من الزمن.

    وكما أشار فاينمان في المحاضرة التي ألقيتها، كان على الأرض أن تتباطأ وتتوقف خلال الـ 4 مليارات سنة التي تدور حول الشمس. لا يحدث ذلك، فهو لا يبطئ حتى.

  82. المعجزات
    إن النظرية النسبية تحد من نطاق الجاذبية ولكنها تفعل ذلك مع الطاقة المظلمة التي تكون في النطاقات الكبيرة أكثر هيمنة من الجاذبية واتجاهها المعاكس.
    جرب مثالا آخر.
    مساء الخير يا معجزات

  83. يهودا عندي سؤالين لك:
    1) هل هناك طريقة لاختبار صحة نظريتك (ربما توقع جيد أو تفسير لظاهرة لا يوجد لها تفسير حتى الآن)
    2) لقد زعمت أن إضافة كتلة داكنة إلى نموذجنا الحالي لحركة المجرات هي طريقة صعبة لجعل نظريتنا تتناسب مع النتائج التجريبية. كيف يمكن إضافة نماذج إضافية (دفع الجاذبية وفرق الضغط) للتحايل على المشكلة التي وصفتها؟ (كما يبدو، أنت فقط الأمور المعقدة)

  84. المعجزات
    ربما سيفاجئك هذا ولكني سأشرحه بالنظرية النسبية العامة. أنا لا أعارضها. إذا كانت هناك مشكلة لا أستطيع تفسيرها باختلافات الضغط مع النسبية ودفع الجاذبية، وكان لا بد لي من إضافة كتلة داكنة، فسأرفع يدي بكل سرور وأعتذر للجميع. وفي الوقت نفسه ليست هناك حاجة للكتلة المظلمة. هات مثالا عمليا وبعد ذلك سنرى إن كنا سنتغلب عليه.
    مساء الخير
    يهودا

  85. إسرائيل شابيرا
    من قال أنه لا يوجد احتكاك في نظام الجاذبية الدافعة؟، هناك احتكاك ولكن لا ينبغي أن يتعارض مع الجاذبية، ببساطة قوة الطرد المركزي للكوكب على سبيل المثال ستكون مساوية للجاذبية الناتجة مطروحًا منها الاحتكاك.
    المعجزات
    لماذا يصعب على الناس أن يفهموا أن الكتلة المظلمة هي التغير في القياسات بحيث تتناسب مع صيغة الجاذبية حتى على مسافات كبيرة من المجرات؟؟
    إيثان
    أعتقد أنه لا توجد جاذبية على المسافات الكونية الكبيرة، وإذا نشأ تناقض من هذا، فيجب على شخص ما أن يتراجع عن نظريته.
    مساء الخير
    سابدارمش يهودا

  86. يهودا
    وبقدر ما أعرف، فإن افتراض النظرية النسبية هو أن الجاذبية لا تتغير مع المسافة.
    الشيء نفسه ينطبق على نظرية الكم.
    أحدهما يرى أن قوة الجاذبية نتيجة لانحناء الفضاء، والآخر يراها كقوة تحملها الجسيمات النظرية، لكن كلاهما لا يشيران إلى تغيرات في الجاذبية على مسافات كبيرة (باستثناء انخفاض التأثير النسبي بالنسبة إلى المسافة بين الجسمين)
    فإذا كنت تزعم أن قوة الجاذبية تختلف عما هو متعارف عليه، أو أن هناك "الجاذبية الدافعة" (أعترف أنني لم أفهم إذا كنت ترى ذلك كقوة إضافية أو مظهر آخر لقوة الجاذبية)، فأنت تجادل ضد هاتين النظريتين ومن المناسب أن تقدم تفسيرات لذلك.

  87. يهودا
    يمكنك الاعتراض على فرضية المادة المظلمة، لكن الادعاء بأن شخصًا ما يحاول تغيير القياسات يعد اتهامًا خطيرًا للغاية. هل تزعم أن هناك مقالاً قام شخص ما بتزويره؟

  88. إلى إيثان
    ولم أنقل الموضوع إلى الأمور الشخصية.
    أنا لا أتنافس مع النسبية ونظرية الكم. أين رأيت ذلك؟
    أنا أعارض الكتلة المظلمة والطاقة وأرى أنهما محاولة لتغيير القياسات التي تم الحصول عليها في المجال بحيث تناسب الصيغة.
    أنت تزعم أيضًا أنني أخالف النظرية الراسخة. هل لي أن أعرف عن أي نظرية تتحدث؟ بخصوص الكتلة والطاقة المظلمة؟؟، آسف، رأيي ليس لك.
    وأما "الحفرة" وغيرها من الصفات، فأعتقد أنه بالغ فيها.
    وأما التعبير عن الرأي بنظام مجهول فهو مجهول، لست كذلك!
    يوم جيد
    سابدارمش يهودا

  89. يهودا
    ليس لدي أي أدوات للرد على تكهناتك. أنا لست فيزيائيًا وينتهي تدريبي في هذا الموضوع بدورة تمهيدية في الفيزياء A وB.
    لكن. أتمكن من اكتشاف التهرب من الإجابة ونقل موضوع المناقشة إلى الخطوط الشخصية.
    إذا كانت لديك نظرية تتنافس مع نظرية المجالات الكمومية والنظرية النسبية، فمن المناسب توفير بعض الأساسات النظرية للنظرية، والإشارة إلى المشكلات الموجودة في النظرية وإلى الملاحظات التي تتحقق منها أو تتعارض معها .
    عند مواجهة نظرية ثابتة فمن المناسب تقديم تفسيرات لتسلسل الفرضيات التي أدت إلى بناء النظرية والإشارة إلى الأسباب التي تجعل المجتمع العلمي لا يقبل هذه النظرية.
    كما أن الجهل في السياق العلمي يشير إلى نقص المعرفة في مجال معين. ليست لعنة كما هو شائع في اللغة اليومية.
    هذه ليست إهانة، ناهيك عن أسباب رفع دعوى قضائية (حتى لو لم تكن تعبيراً عن الرأي في نظام التعليقات المجهولة)

  90. يهودا،

    "النزول إلى مستواي" هل يعني حقاً التعامل بموضوعية مع حقيقة أن النظرية التي تحاول بيعها للجميع تقدم ببساطة تنبؤات يوجد في المختبر أنها غير صحيحة؟

    انت متجاهل والتهديد برفع دعاوى قضائية غبية لن يساعد أحداً ولن يغير حقيقة أنك تتحدث عن أشياء ليس لديك أي فكرة عنها. أنت لا تعرف ما الذي تتحدث عنه، لذا فأنت جاهل بكل معنى الكلمة.

    التهديد بالدعاوى القضائية لا يؤدي إلا إلى زيادة الطين بلة. كما ترى، بغض النظر عن مدى عدم إعجابك بلغتي (أعتقد أن السبب الحقيقي وراء حماسك الشديد ليس لغتي، بل محتوى كلماتي - فأنت تعلم أنه ليس لديك ما تجيب عليه عندما تتعرض للخطر) كالهراء الذي تم فضحه منذ أكثر من قرن من الزمان)، لم أحاول أبدًا إسكاتك. لم أحاول أبدًا إبعادك. أنا لم أهددك أبدا. كل ما فعلته هو التأكد من أن كل شخص يقرأ "تخميناتك" يعرف أنها من الناحية العلمية كومة كبيرة من الهراء. أنت تحاول إسكاتي، وهذا يظهر أشياء عنك أسوأ بكثير من مجرد الجهل.

  91. ألبينتيزو,
    بوضوح. عادةً ما يكون من الوقاحة القفز بسؤال إلى أعلى الموضوع ولكن بسبب "الحمل" قررت أن أغمره مرة أخرى.

    شكرا مقدما

  92. للجميع
    لن أجيب على ألبانتيزو بعد الآن، لا أريد أن أنزل إلى مستواه. من أجله، فليكن حذرًا جدًا في كلامه حتى لا أقاضيه.
    إن وصفي بـ "الجاهل" وما شابه ذلك قد لا يمر بصمت.
    يهودا

  93. يهودا،

    أولاً، أؤكد لك أنني لست صلباً. إيتان محق بالفعل فيما يقوله لك، لكنه يغفل النقطة الأساسية: أنت شخص جاهل (وليس هناك إهانة هنا: أنت نفسك تعترف أنك لم تدرس أبدًا موضوع الجاذبية بما يتجاوز صيغة الصف التاسع، أي ، ليس لديك أي فكرة عن النسبية العامة، في نظرية المادة المظلمة وغيرها) الذي يحاول فرض جهله على الواقع..

    وبالطبع ماذا سيقول الجاهل والجبان عندما يضعون أمامه الرد على هذا الهراء الذي يقوله؟ "لن أتناول ذلك. مللت". بالطبع لن تستجيب لأنك لا تستطيع الرد لأنك تصر وتجادل حول صحة نظرية لا يتناسب عدد تفنيدها في رد أو في عشرة ردود في الموقع. وأنت تصر على ذلك بسبب جهلك. إذا كنت تهتم بدراسة الأشياء التي تتحدث عنها، فلن تقول ما تقوله. كما ترون، بدلا من قبول التفسير الواضح (أن السبب الذي يجعل كل من يفهم الجاذبية يرفض هذه النظرية هو أنه بمجرد دراسة مجمل الأدلة والحقائق ترى أن النظرية ببساطة لا تصف واقعنا)، فإنك تلجأ إلى التفسيرات المذعورة ("أنا الوحيد الذي فكر في هذا! الفيزيائيون المحترفون والناجحون لا يستطيعون فهم ما أقول! هناك مؤامرة! هناك مؤامرة!")، وهذا لأنك رجل جاهل وفقير الذي يعيش في حالة إنكار. وللدليل، تعتقد أن الخبرة في إدارة أعمال تحسين المصانع تؤهلك للحديث عن موضوعات متقدمة في الفيزياء الحديثة. خلاص مفيش وهم أكتر من كده.

  94. إلى المستجيب الجديد إيتان

    في أحد ردودك فعلت ما يلي:
    1. لقد بدأت بالتقليل من شأنك-
    الجواب:- باستثناء واحد، الجميع يستجيب باحترام. الموافقة على رأي الآخر أم لا، فهذا أمر مشروع.
    2. لقد ذكرت أنك تتجاهل عاملاً يخلق مشكلة لفرضيتك دون توضيح سبب تجاهلك له
    الجواب: بعد أن شرحت ذلك عدة مرات، سأشرحه مرة أخرى، اذهب يا إيثان. ففي جاذبية النظام الشمسي، على سبيل المثال، تساوي قوة الجاذبية قوة الطرد المركزي للكوكب. لكن قوة الجذب ليست الجاذبية فقط بل هي توازن الجاذبية والإشعاع الشمسي و... الاحتكاك. ففي نهاية المطاف، تتحرك الجسيمات المختلفة دائمًا في الفضاء. فهو لم يمنع الكواكب من الدوران أبدًا، ولا يوجد سبب لتوقفه إذا كان التفسير هو في الواقع الجاذبية الدافعة.
    3. لقد زعمت أنك لن تحاول حتى الشرح لأن أحد المعلقين هنا، في رأيك، غير قادر على فهم الشرح-
    الجواب: - صحيح، في رأيي أن المجيب لا يريد أن يفهم أو أنه غير قادر على الفهم.
    4. أنت تقر بأن فرضيتك فقدت صحتها في بداية القرن العشرين ولكنها لا تعالج هذا الأمر
    الجواب: على العكس من ذلك، أعتقد أن الوقت قد حان للتحقق من مثل هذه الأفكار كل بضعة سنوات أو أشهر.
    أقوم أيضًا باختبار أفكار أخرى تم قبولها منذ عقود، والتي قد تفاجئك، على سبيل المثال أن سرعة الضوء ليست ثابتة، وأن القمر يتحرك بعيدًا 25 ملم فقط في السنة وليس 38 ملم، وأن وزن الأجسام تتغير كدالة للزمن وبالطبع لا توجد كتلة مظلمة وطاقة. فماذا لو قال الآخرون خلاف ذلك؟
    5. لوم الآخرين على لومك على مشاكلهم
    الجواب: - لقد قال حكماؤنا: إن الناقض في نفسه ناقض. أي عندما تتهم أحداً بشيء ما، راجع نفسك أولاً.
    6. وأخيرًا، تشير إلى نفسك بصيغة الغائب-
    الجواب: لم أر أين، وإذا كان الأمر كذلك فلماذا هو مهم جدا؟
    إذا عرض شخص ما حججه أمامك بهذه الطريقة، فهل سيكون من السهل عليك أن تأخذها على محمل الجد؟
    أو ربما تطلبين منه عدم السماح لاعتبارات الأنا بالتأثير على طريقة تقديمها؟
    الجواب - لم أقل شيئاً دون تفكير قط عندما تكون اعتبارات الأنا فقط هي التي ترشدني، ففي العلم الأنا لا تحدد. في عملية تبسيط الأعمال التي شاركت فيها، كنت أقترب من معظم العمال العاديين في عملية الإنتاج وأسألهم عن رأيهم في المشكلة وسوف تتفاجأ، هؤلاء العمال "الأغبياء" عادة ما يعرفون ما هي المشكلة ويقدمون أفكارًا جيدة للحصول على حل. اعتبارات الأنا الخاصة بي بعيدة جدًا جدًا عني.
    أتمنى أن أكون قد أجبت على أسئلتك
    وأتمنى ألا تكون ألبانزو (على الرغم من أهمية ذلك)
    اسبوع جيد
    سابدارمش يهودا

  95. شموليك،

    في بعض الأحيان لا أتواجد لبضعة أيام (أو بضع ساعات) وفجأة تتراكم الكثير من التعليقات، لذلك لا أملك الطاقة لتعويض كل ما فاتني (أو أريد ذلك). في المستقبل سأعود وألقي نظرة على ما طلبته، ونأمل اليوم (ولكن لا وعود).

  96. يهودا،

    1. إن فهمك للقراءة مجرد مزحة. ومن المؤسف أن يصل الإنسان إلى مرحلة الشيخوخة دون أن يتمكن من فهم نص بسيط. لا تقلق، سأشرح بالضبط ما لم تفهمه (على الرغم من أن المشكلة هنا هي أنك بالطبع لن تفهم ما أشرحه حتى الآن):

    2. لم أقل في أي وقت من الأوقات أنك غير مدرك للاحتكاك. كنت أشير فقط إلى حقيقة أنك زعمت أنه لا يمثل مشكلة. إذن ما يجب فعله يا يهودا يمثل مشكلة. عندما تعطي النظرية تنبؤًا ثم تذهب إلى المختبر، وتسجل الملاحظات، وترى أن التنبؤ لا يتحقق بشكل واضح، فهذه مشكلة. ربما ليس في عالم الخيال للمتقاعدين الذين لديهم معرفة فيزياء الصف الثامن، ولكن في عالم العلوم - مشكلة.

    3. من المضحك أن تكتب "حتى ريتشارد فاينمان كتبه!" ومن ثم ينسب إلي كما لو أنني أعطيت أهمية خاصة لكلمات فاينمان بينما في الواقع كتبت بالأبيض والأسود أن فاينمان ليس سوى مثال (وهو ما أستخدمه لأنك طرحته)، ولكن في الواقع أي شخص يكلف نفسه عناء البحث عن يعرف الموضوع في المائة عام الماضية أنه ببساطة لا يتوافق مع الملاحظات.

    4. إذا كنت تصر على عدم إثبات كلماتك، ولكن ببساطة تكرر مثل الشعار الهراء السخيف (السخيف بالمعنى الرياضي) بأن الاحتكاك ليس مشكلة، فهذا يظهر أنك تعتقد أن هناك ديمقراطية هنا. إن حقيقة قولك "من حقي وأستطيع أن أختلف على أن الاحتكاك مشكلة" تشير إلى أنك تعتقد أن كل شخص لديه رأي وكل رأي له قيمة، وهي الديمقراطية بشكل أو بآخر. ما يجب القيام به، في العلم ليس كذلك. إذا قلت شيئا دون أن تتمكن من تقديم دليل عليه، فلا قيمة لرأيك. في حال لم يكن الأمر واضحا بما فيه الكفاية، سأقول ذلك مرة أخرى - يهودا، رأيك ليس له قيمة. وهذا ليس لأن اسمك ليس Feynman، بل ببساطة لأنك تتحدث فقط عن هراء وليس لديك القدرة على إثبات هراءك كما يتطلب المنهج العلمي: باستخدام نموذج رياضي يوفر تنبؤات تتوافق مع الواقع. ما يجب القيام به، فإن توقعات النموذج المعني تتعارض تمامًا مع الواقع. لقد دعوتك في الرد السابق وأدعوك مرة أخرى - أرجو أن تقدم نموذجًا رياضيًا للتوراة المذكورة لا يحدث احتكاكًا (أي لا يتنبأ بإشارات غير موجودة).

    5. "لن أحاول أن أشرح" = أنا جبان وغبي للغاية، لذا سأختلق عذرًا يفترض أنه يعفيني من تبرير كلامي. ونعم، يهودا، بالطبع لا أستطيع أن أفهم. بالطبع... لماذا يفهم الفيزيائي المحترف (وحتى لو جاز لي أن أشهد، متفوقًا في مجاله) الذي كان يبحث في الجاذبية لسنوات شيئًا تافهًا في الميكانيكا الكلاسيكية؟ من الواضح أنك تحتاج إلى ما لا يقل عن 40 عامًا من الخبرة في إدارة أعمال تحسين المصانع لفهم الحجة الكلاسيكية المتعلقة بـ "لماذا لا يهم الاحتكاك".

    6. ومرة ​​أخرى نعود إلى حقيقة أنه لا يمكنك القراءة على مستوى الصف الخامس. أنت تتهمني بعدم فهم سبب تناسب القوة الميكانيكية مع الكتلة، بينما كتبت في ردي *صراحة* تفسيرًا لذلك. قل هل أنت غبي أم ماذا؟ لقد كتبت شرحًا لهذا على وجه التحديد، وما يجب فعله - فهو لا ينجح إلا في حالات معينة (وهي الحالات التي كانت معروفة قبل مائتي عام، عندما كتبت هذه النظريات). لكن هذا التفسير لا ينطبق على الجسيمات التي ليس لها كتلة ولا شحنة، رغم أننا نرى في الملاحظات أنها تتأثر بالجاذبية (الملاحظات، تذكر؟ ذلك الشيء الذي لا ينبغي أن تتفق معه التوراة في عينيك).

    7. لم أقلل من شأن شانكار أبدًا. كالعادة، عقلك يتحدث هراء. لقد قللت من شأن *أنت*. لقد استهنت بحقيقة أن هناك أحمق في العالم يعتقد أن الحصول على شهادة في الإدارة وخبرة في تبسيط المصانع يجعله مؤهلاً للتعامل مع المادة المظلمة. لدي أصدقاء جيدون درسوا في شانكار، وهم مصممون ممتازون. أنا أحتقرك بقدر ما أحتقر شخصًا درس علم الأحياء وعلم الأعصاب في معهد ماساتشوستس للتكنولوجيا ويعتقد أن ذلك يؤهله لدراسة المادة المظلمة. أنا لا أحتقر علم الأحياء أو معهد ماساتشوستس للتكنولوجيا، بل أنظر باستخفاف إلى الشخص الذي يعتقد أنه من الممكن البحث في الموضوع X دون دراسة الموضوع X. ومن الواضح أن حالتك أسوأ، لأن "مؤهلاتك" ليست حتى أكاديمية. يعني لمصلحة عالم الأعصاب الذي ذكرته سابقاً، عليه أن يقول على الأقل أنه يفهم ما هو البحث، وأن لديه خبرة في دراسة الطبيعة، وطرح الفرضيات ودحضها، وإجراء التجارب، وقراءة المقالات، وما إلى ذلك.

    8. أنا لا "أقول" أنك لا تفكر بشكل منطقي. أنا أعرض بالضبط ما هو غير منطقي في مزيج الهراء الذي تلفظه. على عكسك، أنا *لا* أتقدم أبدًا بادعاء وأرفض دعمه بالأدلة أو التفسير. أتهمك بالغباء وعدم العقلانية لأني أكشف في كل رد أنك تقول هراء.

  97. يهودا
    في أحد ردودك فعلت ما يلي:
    1. لقد بدأت بالتقليل من شأنك
    2. لقد ذكرت أنك تتجاهل عاملاً يخلق مشكلة لفرضيتك دون توضيح سبب تجاهلك له
    3. زعمت أنك لن تحاول حتى الشرح لأن أحد المعلقين هنا برأيك غير قادر على فهم الشرح
    4. أنت تقر بأن فرضيتك فقدت صحتها في بداية القرن العشرين ولكنها لا تعالج هذا الأمر
    5. لوم الآخرين على لومك على مشاكلهم
    6. وأخيرًا، تشير إلى نفسك بصيغة الغائب

    إذا عرض شخص ما حججه أمامك بهذه الطريقة، فهل سيكون من السهل عليك أن تأخذها على محمل الجد؟
    أو ربما تطلبين منه عدم السماح لاعتبارات الأنا بالتأثير على طريقة تقديمها؟

  98. إلى ألبينزو

    في بعض الأحيان يكون من المزعج أن يقرأ شخص مسؤول مثلك الأشياء في لمح البصر، لأنه من الممكن إلقاء الأوساخ على يهودا والتقليل من شأن المعلقين بهذه الطريقة دون التعمق.
    إذا تعمقت قليلاً في ردي، فسوف ترى أنني على دراية بالاحتكاك ولكن لا أعتقد أن هذه هي المشكلة.
    وسوف تتفاجأ، إنه مسموح لي وقادر على أن أختلف في أن الاحتكاك مشكلة، على الرغم من أن شخصًا عظيمًا وعظيمًا مثل ريتشارد فاينمان قال ذلك! وليس هناك أي معنى للديمقراطية هنا.
    لن أحاول أن أشرح لك لماذا الاحتكاك ليس مهما لجاذبية الجاذبية الدافعة لأنك لا تريد أن تفهم وهو مخالف لتصورك أن تفهم وربما لا تستطيع أن تفهم أيضا!
    لن تتمكن أيضًا من فهم سبب تناسب تصادم الجزيئات فعليًا مع الكتلة وليس مع السطح، لأنك لا تريد أن تفهم، ومن المؤسف أنه رغم كل شيء لديك الكثير من المعرفة و رأس سليم على كتفيك كان من الممكن أن يساهم.
    هل تعتقد أنني لا أقرأ أن النظرية قد تم دحضها منذ سنوات بالفعل؟؟، ولكني تعلمت من شانكر (تذكر؟؟، ذلك الطائر الذي كنت تحتقره بشدة؟؟) وحتى قبل ذلك من معلمة الروضة إستر في المدرسة روضة "بامبي" أنه إذا لم تتعاملي مع موضوع معين لعدة سنوات فيجب أن يكون الوقت قد حان للعناية به"، فأنت تتحدث عن مائة عام لم يبتكروا شيئًا في هذا الموضوع من دفع الجاذبية؟؟
    لكنك بالطبع تقول إنني لا أفكر بشكل منطقي، ولا أريد أن أفهم، وما إلى ذلك، وما إلى ذلك. يا رجل، هل تعلم أن الجمل لا يرى سنامه؟؟

    كل التوفيق لك ولجميع شعب إسرائيل
    اسبوع جيد
    يرجى الرد بلطف.
    يهودا

  99. ليوفال خالكين
    لست متأكدًا من وجود مثل هذه القائمة لوالدي، فحتى أنا القديم أحيانًا ينتظر بصبر تأكيد الرد
    ربما يكون من المرغوب فيه حقًا أن يكون لديك مثل هذه القائمة!
    فقط بعد أن "يتسخ" شخص ما، قم بفرض الرقابة عليه.
    اسبوع جيد
    يهودا

  100. إسرائيل شابيرا

    من المسلم به أنه نعم، حتى مع الغاز العادي، من الممكن إظهار الجاذبية وفقًا لمبادئ دفع الجاذبية. للحصول على تفاصيل حول خيار القياس اتصل بالبريد الإلكتروني الخاص بي

    sevdermishy@gmail.com
    اسبوع جيد
    سابدارمش يهودا

  101. 1. لم أقرأ إدخال ويكيبيديا، لكنني رأيت مرجع فاينمان. ما يتحدث عنه هناك هو بالضبط النظرية الميكانيكية للجاذبية عند ليساج وآخرين.

    2. "فاينمان يلغيها بسبب الاحتكاك، وهو ما لا أتفق معه." ما هذه يا ديمقراطية؟ ماذا يعني "لا أوافق"؟ إذا كان فاينمان مخطئًا، فأثبت ذلك! (بالمناسبة، هذا ليس فاينمان. قبل خمسين عاما، أدرك الناس أن هذه الفكرة ببساطة لا تتوافق مع الواقع بأي شكل من الأشكال). من فضلك شاهد نموذج الجاذبية الميكانيكية كما هو موضح في الفيديو والذي *لا* ينتج عنه قوى احتكاك.

    4. بالطبع الاحتكاك ليس هو المشكلة الوحيدة، هو ببساطة شيء تافه حقا يمكن تفسيره في سطر واحد ونرى بوضوح أن النظرية تسقط لأنها لا تتناسب مع الواقع. هناك العديد من التناقضات بين الفكرة المعنية وواقعنا. على سبيل المثال، في النظرية الميكانيكية، تتناسب الجاذبية مع مساحة سطح الجسم الذي يعاني من الجاذبية وليس مع الكتلة. وقد جرت العادة على حل هذه المشكلة بالقول إن الجزيئات الموجودة في التدفق صغيرة جدًا وبالتالي تخترق الجسم فعليًا وتتفاعل مع الوحدات البنائية التي لها كثافة ثابتة وبالتالي تتلقى نسبة إلى الكتلة. لكن هذا ببساطة غير صحيح في المقاييس المجهرية، وبطريقة أبسط، ولا ينطبق على الجسيمات عديمة الكتلة. علاوة على ذلك، نحن نعلم اليوم أنه لا يوجد شيء اسمه "القوة الميكانيكية" - فكل القوى الميكانيكية هي نتيجة متوسط ​​التفاعلات الكهرومغناطيسية. وبالتالي، إذا فهمنا تدفق الجسيمات على أنه يمارس قوة ميكانيكية، فإنه لن يتفاعل مع الجسيمات الأولية غير المشحونة (لنفس السبب الذي يجعل الفوتون لا يمكن دفعه). نحن نعلم بالتأكيد وبشكل قابل للقياس أن الجسيمات غير المشحونة و/أو غير الضخمة تشعر بالجاذبية. والقائمة تطول وتطول…

    ولكن ماذا تفعل، عندما يأتي رجل يرفض صراحة دراسة موضوع معين قبل أن يتحدث عنه، ويحاول أن يبيع على المواقع الشعبية نظريات تم دحضها تجريبيا منذ أكثر من قرن من الزمان، فمن الصعب أن نتوقع منه أن يستمع إليه. مثل أشياء تافهة كالحقائق أو المنطق، أو اتخاذ الحد الأدنى من الخطوات لفهم ما يتحدث عنه، فمن السهل إثبات ادعاءاته (أو على الأقل محاولة إثباتها. من المستحيل إثباتها، لأنها خاطئة).

  102. الجاذبية هي عملية تحدث في أي مساحة تسكنها المادة المظلمة وتتجلى في ظاهرة انجذاب الأجسام لبعضها البعض. كان نيوتن قد توصل بالفعل إلى صيغة تربط خاصية الأجسام، المعروفة بالكتلة، بالمسافة النسبية بينها وبين التسارع الذي تقترب به من بعضها البعض. من الناحية النظرية، هذه الصيغة جيدة وصحيحة لأي كتلة وأي مسافة - الذرات الفردية مثل مجموعات المجرات، ولكن داخل الهياكل التي تسمى "الذرات" يتم الكشف عن انحراف عن الصيغة، وهذا يتطلب فهمًا جيدًا لكيفية إنشاء كتلة السمة من المادة المظلمة.

  103. الى كل مان ديباي
    لقد قرأت نظرية Le Sage حيث تم شرح كيف يرى تكوين الجاذبية وكيف يختلف عن طريقتي. وربما لهذا السبب استنتج أن سرعة الجسيمات يجب أن تكون 100,000 ضعف سرعة الضوء.

    مسار كثيرين آخرين وطريقي تم شرحه في محاضرة ريتشارد فاينمان في جامعة كورنيل بدءاً من الدقيقة الثامنة.
    https://www.youtube.com/watch?v=kd0xTfdt6qw

    • لاحقاً، رفضها ريتشارد فاينمان بسبب الاحتكاك، وهو ما لا أتفق معه.
    أي شخص يريد أن يلقي نظرة.
    ويوفال هانكين، إذا لم تكن لدينا مشكلة مع درجة حرارة القهوة، فمن المحتمل أنك موضع ترحيب. مع السلامه!
    اسبوع جيد
    يهودا

  104. شكرا يهودا
    أحب قهوتي باردة 🙂
    أثناء الانتظار الطويل حتى تذوب تعليقاتي، اكتشفت أنه من الواضح أن هناك عالمًا موازيًا على فيسبوك وأوصي بأن نقضي جميعًا وقتنا هناك أيضًا
    وداعا (ي)

  105. مرحباً يوفال تشيكين
    كيف حالك؟، أنت بالطبع لا تزال مدعوًا لتناول القهوة ولكن ضع في اعتبارك أن الجو يزداد برودة لأن الكون كله يبرد وبالإضافة إلى ذلك فإن ثابت الغرور والتوسع (المتسارع!) للكون يجعلان اللقاء بيننا أكثر صعوبة.
    كل خير
    سابدارمش يهودا

  106. إسرائيل شابيرا

    يودا، دعونا نجرب هذا:
    1. إذا كان متوسط ​​مسافة الجزيئات عن بعضها البعض كبيرا واصطدمت ببعضها البعض - فسنحصل على الجاذبية، ولكن على مسافات كبيرة فقط.
    وهذا يتناقض مع ما نعرفه عن الجاذبية حتى على مسافات صغيرة.

    الجواب: لا!!!، فقط على مسافات صغيرة ستحدث الجاذبية!. إن اصطدام الجزيئات فيما بينها يجعل حركة المصادمات عشوائية وبالتالي تصبح المعلومات الموجودة فيها على مسافات كبيرة عشوائية.

    2. إذا كانت المسافة صغيرة، حصلنا على غاز بسيط.
    في مثل هذه الحالة لن يكون هناك جاذبية على الإطلاق، لأنه لن يكون هناك فرق في الضغط، كما لا توجد جاذبية بين جسمين في الهواء.
    الجواب: أنت مخطئ مرة أخرى. بين جسمين متقاربين (عدة سم) في الهواء لن تحدث جاذبية لأن متوسط ​​المسار الحر لجزيئات الغاز صغير جدا في حدود الميكرومترات وبالتالي تفقد الجزيئات المعلومات عن الجاذبية التي تحملها في حركتها الموجهة . إذا كان الغاز أرق وأصبح متوسط ​​المسار الحر يبلغ حجمه عشرات السنتيمترات على الأقل، فسيظهر السحب.

    فيما يتعلق بالقسم 3، لا أستطيع أن أشرحه لك بشكل أفضل ولسوء الحظ سيتعين علينا أن نترك شيئًا للاجتماع بيننا.
    يبدو لي أننا نفهم دفع الجاذبية بشكل مختلف عن بعضنا البعض

    انا انتهيت
    الذهاب لتناول الطعام والراحة. باي
    يهودا

  107. يهودا،
    النموذج القياسي لا يحتوي على مادة مظلمة. ربما يجب عليك أن تقرأ قليلا عنه.
    تخبرك نظرية المجال ما هي الطاقات التي سيكون من الممكن تحديد موقع جسيم محتمل لم يتم اكتشافها بعد. خمين ما؟ مثل هذا الجسيم غير المكتشف إما بالكاد يتفاعل أو سيكون مداه أقصر بكثير من نطاق القوة النووية القوية. يسمى هذا الجسيم بالمادة المظلمة ولن يفعل ما تريد أن يفعله

    بالإضافة إلى ذلك، أنت مرتاح جدًا للجزيئات التي تبلغ سرعتها 1.3 مرة أكبر من سرعة الضوء. أنا آسف، الأمر ليس خطيرًا

  108. للمعجزات والمعجزات

    حصلت على نتائج مختلفة عن ويكيبيديا. وتبلغ سرعتها النيوتونية 1.3 مرة سرعة الضوء وهي في الحساب النسبي قريبة من سرعة الضوء، وكتلة الجسيم تقترب من عشرة أس سالب 36 جراما.
    ولكن، سأكون لطيفًا أيضًا، وإذا كنت أنت ونيسيم سعيدين بالنموذج القياسي حيث يمكن للمرء دائمًا إضافة الكتلة (الداكنة) بقدر ما هو مطلوب وبقدر ما هو مفقود والحصول على أي نتيجة مطلوبة، فأنا سعيد بذلك سعادتك!
    على أية حال، سأبحث عن المقالة التي كتبت فيها استنتاجاتك حول سرعة وكتلة الجسيم ثم سأرد مرة أخرى. وفي هذه الأثناء، كل شيء على ما يرام.
    يرجى الرد بلطف
    يهودا

  109. يودا، دعونا نجرب هذا:

    1. إذا كان متوسط ​​مسافة الجزيئات عن بعضها البعض كبيرا واصطدمت ببعضها البعض - فسنحصل على الجاذبية، ولكن على مسافات كبيرة فقط.

    وهذا يتناقض مع ما نعرفه عن الجاذبية حتى على مسافات صغيرة.

    2. إذا كانت المسافة صغيرة، حصلنا على غاز بسيط.

    في مثل هذه الحالة لن يكون هناك جاذبية على الإطلاق، لأنه لن يكون هناك فرق في الضغط، كما لا توجد جاذبية بين جسمين في الهواء.

    3. إذا كانت الجزيئات لا تتصادم ولا تتداخل وعموما لا تتداخل مع بعضها البعض وكانت المسافة بينها صغيرة - فسنحصل على الجاذبية حسب نموذج لاساج عند أي مسافة.

    ولكن بعد ذلك نعود إلى سؤال فاينمان الأساسي، الذي كنت أطرحه عليك منذ سنوات: ماذا عن الاحتكاك؟

    لا يمكنك استبعاد هذا السؤال مع وجود مسافة كبيرة بين الجزيئات، لأننا بعد ذلك نعود إلى السيناريو 1، أي أن الجاذبية لن تنشأ على مسافات صغيرة، وهو ما يخالف ما نعرفه من النتائج التجريبية.

    فكيف يعمل؟

  110. ألبينتيزو,
    إذا كان لديك صبر، فعندي مسألتان أحب أن أسمع رأيك فيهما.
    يتحدث أحدهم عن المنشور التالي لشون كارول (مع اعتذاراتك لأنني أستمر في اقتباس كلماته):
    http://www.preposterousuniverse.com/blog/2014/05/05/squelching-boltzmann-brains-and-maybe-eternal-inflation/

    كنا نتحدث في ذلك الوقت عن برنامج لهيئة الإذاعة البريطانية (BBC) قدم بعض الأفكار حول ما حدث قبل الانفجار الكبير وأحدها كان لروجر بنروز الذي قال أنه عندما يعرف كل شيء فإن الكون "سيفقد" أبعاده ثم يعاد خلقه بسبب ذلك. للتقلبات الكمومية (آمل ألا أكون مخطئًا هنا!). هل ترفض الدراسة التي أجراها كارول وزملاؤه هذه الفكرة بالفعل؟ أليس الفراغ بدون راصد غير مستقر؟ ما هو بالضبط المشاهد؟ هل لدى المشاهد أي *مادة* كلاسيكية؟
    كما أنه يرتبط أيضًا بكتاب لورنس كراوس وهوكينج "الكون من لا شيء". يبدو أن كارول يرفض هذه الفكرة.

    الموضوع التالي عن البروفيسور . وقال جيمس جيتس في ندوة نوبل وأيضا في الرابط المرفق عن أكواد تصحيح الخطأ الموجودة في نظرية الأوتار.
    http://youtu.be/cvMlUepVgbA
    هل تعرف الموضوع؟

    شكرا مقدما

  111. يهودا،
    أنا آسف يهودا، لكن مقترحات الحل التي قدمتها تافهة للغاية، لكي أكون لطيفًا.
    TeVeS هي محاولة لإنتاج شيء نسبي لكنني أرسلتك إلى محاضرتين حيث أخبروك أنه لا يزال لا يحل المشكلة.
    يخبرك نيسيم أنه وفقًا للنموذج القياسي (الذي حقق نجاحًا مذهلاً في توافقه مع الواقع) لا يمكن أن تكون جسيمات الجاذبية الدافعة جسيمات معروفة، ووفقًا لموقع ويكي، يجب أيضًا أن تكون أسرع من الضوء (ولكن عند بعض السرعة المحدودة، وهو ما يتعارض مع النظرية). النسبية التي أردت الحفاظ على الجاذبية) وأنت تجيب باستخفاف شديد. لا يعني ذلك أنه من الصعب اكتشافها، ولكن وفقًا للنموذج القياسي فهي غير موجودة. إذا كانت موجودة، فهي إما ثقيلة جدًا جدًا جدًا وبالتالي فإن نطاق عملها قصير جدًا جدًا جدًا أو أنها بالكاد تتفاعل مع المادة العادية وبالتالي لا يمكنها فعل ما تريد منها أن تفعله.
    بالمناسبة، تسمى الجسيمات التي يصعب تمييزها بالمادة المظلمة

  112. المعجزات

    لا أتفق مع افتراضاتك:
    وهي مكونة من جزيئات لها زخم وطاقة حركية.
    فهي صغيرة جدًا ويصعب ملاحظتها.
    وفكرة دفع الجاذبية لا تستبعد النسبية.

    ليلة سعيدة الذهاب إلى السرير
    وداعا المعلقين من جميع أنحاء العالم!
    وداعا الكون!
    شكرا "يادن"!
    ()

  113. على الرغم من أن النماذج الفيزيائية تتوافق بشكل جيد مع الرياضيات، إلا أنني لم أتلق حتى الآن إجابة على سؤال حول الفيزياء وفواتير البقالة. أي كيف يحدث أن تلعب الرياضيات دورًا في هذه النماذج.
    المتدينون بيننا (من حسن الحظ أنه السبت الآن ولن يدنسوه لكي يضربوني ويضربوني) ينسجمون جيدًا مع عالم مزدوج حيث الرياضيات والعناية السماوية في خضوع واحد. ومن ناحية أخرى، أبحث دائمًا عن الحلول البسيطة. لا أحب، على سبيل المثال، أن يكون عالمنا مأهولًا بالعديد من الجسيمات الأولية التي ليس لها سلف مشترك. على الرغم من أن الفيزيائيين المعاصرين حاولوا استرضائي بنموذج الكوارك، إلا أن ستة أيضًا رقم كبير جدًا بالنسبة لرأيي، فماذا عن حقيقة أن كل كوارك من الكواركات لديه كوارك مضاد وقد وصلنا بالفعل إلى عشرات الوحدات البنائية المنفصلة .
    النموذج الذي كنت أعمل عليه لمدة 40 عامًا (سيكون عيد ميلادي قريبًا) يتحدث عن جسيم أولي واحد فقط والذي اتخذ بالفعل في أيامه الأولى شكل ما يعرف اليوم باسم "المادة المظلمة".
    المادة المظلمة هي التي تشكل الجسيمات المعروفة للفيزياء. وهو أيضاً من خلق ظاهرة الجاذبية من جهة والكهرومغناطيسية من جهة أخرى.
    سيكون من الخطأ أن نعزو خصائص فيزيائية مألوفة للمادة المظلمة، لأن الفيزياء مستمدة منها، ومثل هذا الإسناد سيشكل تعريفًا دائريًا. ولذلك فإن الحديث عن "الفوتون المظلم" على سبيل المثال هو مجرد هراء وأنا أؤيد بشدة سيفا المقال "أوبل 3827 سوف يصبح مثالا آخر على ما ليست المادة المظلمة".

  114. إسرائيل شابيرا
    لن تكون هناك جاذبية في الهواء لأن متوسط ​​المسار الحر للجزيئات (الجزيئات) فيه صغير جداً وبالتالي لن يحدث أي تجاذب. لكن في ظل ظروف الفراغ العالي سيكون من الممكن إجراء تجربة تظهر الجاذبية، لأنه عندها يزداد متوسط ​​المسار الحر. هل تفهمون إسرائيل، فمن يريد الاستثمار في التجربة؟

  115. يهودا
    كما أفهمها، لا يمكن أن تكون مصنوعة من مادة عادية، لأنها ستظهر في النموذج القياسي.

    وأعتقد أيضًا أن عدم البحث عنهم لا يعني أنه لم يتم العثور عليهم.

    وشيء أخير: الفكرة تستبعد كلا النظريتين النسبيتين. لا يبدو لي أن الأدوات التي انتقلت منذ فترة طويلة من مجال العلوم إلى مجال الهندسة يجب أن يتم استبعادها بهذه السرعة.

  116. يودا

    إذا كان هناك تجاذب في آلية غازية مع تصادمات بين الجزيئات، فسنحصل على تجاذب بين أي صفيحتين ممتدتين تواجهان بعضهما البعض، أليس كذلك؟

    لاحظ أنه بدون الاصطدامات، ستحصل على الجر كما هو موضح بالقيمة الموجودة في Lasage.

  117. إسرائيل شابيرا

    لماذا حددت أن قوى الجاذبية لا تتولد في آلية غازية؟؟؟، وأنا أسعى منذ سنوات لإجراء تجربة تظهر قوى الجاذبية في الغاز. أبحث عن متطوعين لإجراء مثل هذه التجربة.
    بالإضافة إلى ذلك، لم يعلق لا سيج على إمكانية تصادم الجزيئات فيما بينها لأنه حينها سيكون قد صحح مربع المسافة لمسافات كبيرة مثلما صححتها. بقدر ما أعرف، أنا الوحيد الذي توصل إلى هذه الفكرة.
    وبالمناسبة، فإن فكرة الجسيم الرسول الذي يسبب الجاذبية قد طرحها نيوتن بالفعل. ادعى أنه يجب أن يكون هناك واحد لكنه لا يعرف كيف يعمل.

    يهودا

  118. المعجزات
    هذه الجسيمات صغيرة الحجم ولها كتلة أصغر من كتلة النيوترينوات. لديهم قوة دافعة وطاقة حركية، وعلى الأرجح أنها مصنوعة من مادة عادية. يبدو أنهم مثل النترينات الصغيرة. أسميهم قطع La Sage الخاصة بي. لم يتم اكتشافهم لأنه لم يتم البحث عنهم. لا يؤمن الكثيرون بدفع الجاذبية. ولكن إذا كنت على حق بشأن اختفاء الجاذبية على مسافات كبيرة، فقد يكون هذا دليلاً على وجود تفسير دفع الجاذبية للجاذبية.

    إلى شموليك
    أنا لست من أتباع نظرية MOND، لكن التجديد يحدث من انتقال بين مناطق ذات كثافة غازية مختلفة موجودة في محيط المجرة وليس بالضرورة بسبب قرب كتلة كبيرة. أعلم أن كيرين أور تفعل هذا مثل بيتا مورجانا أو ميراج، لكني لا أعرف كيف يتم ذلك بالضبط.
    بالإضافة إلى ذلك، لم أكن أتحدث عن جسيمات أسرع من سرعة الضوء ولم أتحدث عن النظرية النسبية. كل ما أفكر فيه بشأن النظرية النسبية هو أنها تحتاج إلى مراجعة، على سبيل المثال فيما يتعلق بالجاذبية على المسافات.
    بصراحة، لم أتعمق في الأمر كثيرًا. ومن جهتي، سيتم الحفاظ على احترام النظرية النسبية. لا ينبغي لتفسير مختلف للجاذبية أن يضر بأساس استنتاجات النسبية.
    يمكنني أن أستمر، لكن الوقت قد تجاوز منتصف الليل بالفعل، لذا يرجى الرد بلطف. الجيران نائمون.
    يهودا

  119. يودا

    وكان يوفال تشيكين يعيش في اسكتلندا في ذلك الوقت، والآن في نيوجيرسي. هو الذي ربطني بك.

    تصحيح بسيط: في Balsage لا يوجد تصادمات بين الجزيئات، وإلا فإنك ستحصل على آلية غازية، والتي كما نعلم لا تخلق الجاذبية.

    أفهم ادعاءك أنه وفقًا لنموذجك، فإن الاصطدامات، جذر متوسط ​​التربيع، تكون على مسافات كبيرة وبالتالي نحصل على آلية غازية، ولكن كما ذكرت لك بالفعل، في هذه الحالة لن تحصل على الجاذبية على مسافات قصيرة، والتي يتعارض مع النتائج التجريبية

  120. يهودا،
    إذن فرق الضغط هو بطاقة خروج مجانية من السجن؟ هل من الممكن دمجها في المحادثة ببساطة دون أن تكون خبيرًا فيما يتم قياسه في مجموعة التعداد النقطي؟

    إذا لم يساعد موقع كارول على YouTube، فربما يساعده موقع Wiki؟
    https://en.m.wikipedia.org/wiki/Bullet_Cluster

    في النظريات التي لا تحتوي على مادة مظلمة، مثل ديناميكا نيوتن المعدلة (MOND)، من المتوقع أن يتبع العدسات المادة الباريونية؛ أي غاز الأشعة السينية. ومع ذلك، فإن العدسة تكون أقوى في منطقتين منفصلتين بالقرب من المجرات المرئية (وربما تتطابقان معها). وهذا يوفر الدعم لفكرة أن معظم الكتلة في زوج الكتلة هي في شكل منطقتين من المادة المظلمة، والتي تجاوزت مناطق الغاز أثناء الاصطدام. وهذا يتوافق مع التنبؤات بأن المادة المظلمة تتفاعل بشكل ضعيف فقط، بخلاف قوة الجاذبية.

    ما زلت لا أفهم كيف يمكن لمكافأة الضغط أن تفسر هذه النتيجة

    كما أنني لا أفهم لماذا لا تمثل الحاجة إلى جزيئات تتحرك بسرعات أسرع بكثير من الضوء مشكلة. كيف يمكن تفسير التأثيرات النسبية من خلال هذه النظرية؟ لماذا يتحرك الوقت بشكل أبطأ عند الطيران بسرعة عالية؟ لماذا يتحرك الوقت بشكل أبطأ تحت الجاذبية؟ كيف يتم شرح السحب الأولي؟

  121. للمعجزات
    أنت تجبرني على التفصيل.. حسنًا، الجاذبية الدافعة تخلق الجاذبية بمربع المسافة بطريقة لطيفة حقًا.. يشرحها ريتشارد فاينمان بشكل جيد في محاضرته في جامعة كورنيل. التفاصيل على جوجل. لكن هناك مشكلتان في هذا، الأولى هي الاحتكاك الذي دفع ريتشارد فاينمان إلى إلغاء التفسير. وأنا أزعم أنه لا شيء لأنه يتطلب تصحيحًا بسيطًا في صيغة نيوتن حيث: قوة الجاذبية ناقص قوة الاحتكاك تعطي القوة الطاردة المركزية. تماما مثل السفن الشراعية في البحر حيث قوة الريح ناقص الاحتكاك ستعطي قوة حركة السفينة الشراعية.
    المشكلة الثانية هي أنه كلما زادت المسافة بين الأجسام، زاد احتمال اصطدام الجزيئات ببعضها، ويصبح اتجاهها عشوائيا وبالتالي تفقد معلومات الجاذبية التي تحتويها في اتجاه حركتها. ولذلك لا توجد جاذبية (تقريبًا) على المسافات الكونية الكبيرة. لمزيد من التوضيح، ربما سأكتب لوالدي مقالًا عن دفع الجاذبية وربما يكون مفيدًا لنشره.

    يرجى الرد بلطف. إنه مجرد علم.
    شاب شالوم
    سابدارمش يهودا

  122. إسرائيل شابيرا، هل أنت في الولايات المتحدة؟؟؟... فكرت أقرب بكثير.... لم أنتهي من البكاء لمدة ساعة. ومن الواضح أن فرصة لقاءنا تبتعد أكثر فأكثر، خاصة وأن الكون بأكمله يتوسع بسرعة مع الطاقة المظلمة!!!!!!!!!!!!

  123. آرييل لا يزعم أن هناك خللاً منطقياً أساسياً في جاذبية ليساج، بل خللاً منطقياً أساسياً في عملية يهودا لاستخلاص النتائج. لقد أعطى رابطًا لمقطع فيديو حيث (من المفترض بالطبع) توصل بشكل منطقي إلى استنتاج مفاده أن المادة المفقودة في المجرات الحلزونية يجب تفسيرها من خلال اختلافات الضغط. هذا الفيديو مليء بالهراء والافتراضات والأخطاء المنطقية المحرجة. وأشار آرييل على وجه التحديد إلى حقيقة أنه من ناحية، فإن استنتاج يهودا هو أنه يجب استبدال الجاذبية - وأن ما نعرفه عنها لا ينطبق إلا على المسافات القصيرة، وبالتالي فمن الممكن في الواقع أن يكون مختلفًا تمامًا عما نعتقد - ولكن على ومن ناحية أخرى فهو يستبعد إمكانية التدخل بالقوات النووية لأنها لا تعمل إلا على المدى القصير ناهيك عن أنه من الهراء تمامًا أننا نقيس الجاذبية على مسافات قصيرة فقط (هناك ظواهر جاذبية يتم قياسها حتى في أكبر صخور الكون)، فمن الواضح أن هناك منطقًا ملتويًا هنا. فمن ناحية - القوة الشديدة (التي تقاس فقط على مسافات قصيرة عبر الأرض) يجب أن تخضع لهذه القوانين حتى على مسافات طويلة وبالتالي لا يمكنها تفسير هذه الظاهرة، ومن ناحية أخرى، تم قياس الجاذبية فقط على مسافات قصيرة لذا فمن الممكن أن يتصرف وفق قوانين أخرى ويفسر الظاهرة.

    منطق يهودا مشوه تماما، والمحكمة لن تساعد. كما أنه يتحدث دون أن يعرف نهاية نهاية الحقائق، وهذا لن يتغير أيضًا (وهو يعترف بذلك تمامًا أيضًا، لكنه يدعي لسبب ما أنه ليس من الضروري معرفة الحقائق للتحقيق في القضية). ليس هناك مشاكل منطقية في جاذبية ليساج، بل هناك فقط مسألة صغيرة تتمثل في عدم اتساقها مع الملاحظات. إنها مسألة صغيرة بالفعل، ولكن لسبب ما، يبالغ الفيزيائيون فيها كثيرًا...

  124. يهودا
    إذن يتم امتصاص هذه الجزيئات - بماذا؟ ماذا يصبحون؟ إذا تم امتصاصها في الفضاء الفارغ، أليس من المنطقي أن يتم امتصاصها أكثر عند المرور عبر المادة؟

  125. يودا

    الولايات المتحدة..

    وقد انسحبوا من المملكة المتحدة بالفعل في عام 1776.

    ومملكة والدي هي العلم، وأنا أتحدث عن الفيسبوك. أرسلت لك الطلب منذ شهرين..

    والقوة الشديدة لا تساوي الكثير خارج حدود الذرة. في الواقع، إذا كنت أتذكر بشكل صحيح، فإنه في مرحلة ما ينعكس ويصبح قوة تنافر بدلاً من قوة جاذبة..

    هل يذكرك بشيء في سياق المقال؟

  126. يهودا كنت أجادل فقط وفقًا لمنطقك (إذا لم نقم بالقياس في المسافات المجرية فهذا ليس صحيحًا بالضرورة) أن هناك فرصة جيدة أن تكون في الواقع قوة أخرى (3/4) تؤثر على حركة المجرات وليس بالضرورة الجاذبية. أريد أن أعرف ما هي حججك حول لماذا يجب أن تكون الجاذبية وليس قوة أخرى.

  127. إلى شموليك
    يمكن لفرق الضغط أن يحرك المجرات في العنقود دون الحاجة إلى كتلة داكنة أو جاذبية على الإطلاق.
    الصور الجميلة التي تبدو وكأنها تظهر الكتلة المظلمة في مجموعة المقلاع، تظهر في الواقع فقط مناطق عدم الاتساق مع صيغة الجاذبية. يمكن لاختلافات الضغط أن تفسر كل ما يمكن أن تفسره الكتلة المظلمة. أبعد من ذلك، أنا لست خبيرا في مجموعة القاذفات.
    يوم جيد
    سابدارمش يهودا

  128. الى ارييل

    إذا أثبتت لي أن القوة القوية و/أو وحش السباغيتي الطائر و/أو الملاك غابرييل يعمل أيضًا على مسافات بعيدة، سأكون سعيدًا باستخدامها. مجرد القول أنه غير مقبول بالنسبة لي. من حيث المبدأ، نظرًا لأن غابرييل والسباغيتي ليسا أفكارًا علمية، فأنا أرفضهما تمامًا. (من المستحيل إظهار الطريقة التي يمكن من خلالها تعريضهم للتفنيد).

    إلى العزيز والحبيب إسرائيل شابيرا

    معاذ الله أن أرتكب مثل هذا العمل الوضيع ولا أوافق على أعز أصدقائي في المملكة المتحدة. إذا كان لدى أي شخص سلطة عدم الموافقة عليك، فهو والدي العزيز. من الواضح أنك كتبت بعض الكلمات "المظلمة" التي لم تمر عبر برنامج الرقابة الخاص به. ولكن هنا هو المظهر. فقط تذكر أنني كل صباح أقوم بتسخين القهوة التي أعددتها لك، والدموع في عيني، أنا الكافر أدعو الله أن تظهر أخيرًا.
    ووفقا لك، أنا من محبي دفع الجاذبية، وأعتقد أنه تفسير جميل للجاذبية، ودعونا لا نذكر الاحتكاك لأنه في الواقع يظهر حجم جزيئات لا سيج الخاصة بي. الذين يعرفون كيفية التغلب على الاحتكاك وخلق قوة الجاذبية. كالسفن الشراعية التي تطفو في البحر رغم احتكاكها بالماء والجو. وهذا ليس تناقضاً على الإطلاق!!!
    إن المناقشة الشاملة لرأيي حول دفع الجاذبية هي خارج نطاق هذه المقالة.

    للمعجزات

    أنت على حق في أن هناك آلية «امتصاص»، لكنها ستوسع النقاش في مسألة دفع الجاذبية، وهو أيضًا في حالة إنكار دائم بسبب كلام المنافسين نيوتن وريتشارد فاينمان وغيرهم.
    إذا وافق المسؤول عن الموقع سأكتب مقالاً عن الموضوع. ولكن تبين لي أنه لا يهتم بالمقالات التي لا توافق الإجماع العلمي. وبالطبع هو الرئيس.

    لذا يرجى الرد بلطف
    إنه مجرد علم
    سابدارمش يهودا

  129. الى ارييل

    إذا أثبتت لي أن القوة القوية و/أو وحش السباغيتي الطائر و/أو الملاك غابرييل يعمل أيضًا على مسافات بعيدة، سأكون سعيدًا باستخدامها. مجرد القول أنه غير مقبول بالنسبة لي. من حيث المبدأ، نظرًا لأن غابرييل والسباغيتي ليسا أفكارًا علمية، فأنا أرفضهما تمامًا. (من المستحيل إظهار الطريقة التي يمكن من خلالها تعريضهم للتفنيد).

    إلى العزيز والحبيب إسرائيل شابيرا

    معاذ الله أن أرتكب مثل هذا العمل الوضيع ولا أوافق على أعز أصدقائي في المملكة المتحدة. إذا كان لدى أي شخص سلطة عدم الموافقة عليك، فهو والدي العزيز. من الواضح أنك كتبت بعض الكلمات "المظلمة" التي لم تمر عبر برنامج الرقابة الخاص به. ولكن هنا هو المظهر. فقط تذكر أنني كل صباح أقوم بتسخين القهوة التي أعددتها لك، والدموع في عيني، أنا الكافر أدعو الله أن تظهر أخيرًا.
    ووفقا لك، أنا من محبي دفع الجاذبية، وأعتقد أنه تفسير جميل للجاذبية، ودعونا لا نذكر الاحتكاك لأنه في الواقع يظهر حجم جزيئات لا سيج الخاصة بي. الذين يعرفون كيفية التغلب على الاحتكاك وخلق قوة الجاذبية. كالسفن الشراعية التي تطفو في البحر رغم احتكاكها بالماء والجو. وهذا ليس تناقضاً على الإطلاق!!!
    إن المناقشة الشاملة لرأيي حول دفع الجاذبية هي خارج نطاق هذه المقالة.

    للمعجزات

    أنت على حق في أن هناك آلية «امتصاص»، لكنها ستوسع النقاش في مسألة دفع الجاذبية، وهو أيضًا في حالة إنكار دائم بسبب كلام المنافسين نيوتن وريتشارد فاينمان وغيرهم.
    إذا رغب آفي بيليزوفسكي، سأكتب مقالاً حول هذا الموضوع. لكن والدي أوضح لي أنه لا يهتم بالمقالات التي لا توافق الإجماع العلمي. وبالطبع هو الرئيس.

    لذا يرجى الرد بلطف
    إنه مجرد علم
    سابدارمش يهودا

  130. نعم، لا أعتقد أن المعلقين على دراية بآلية دفع الجاذبية..

    ومن الجدير بالذكر أن الفكرة تم اختبارها من قبل جميع العظماء، بما في ذلك نيوتن، وماكسويل، واللورد كلفن، وفاينمان وغيرهم الكثير.

    ولماذا لا توافقني على الفيسبوك؟ هل نسيت بالفعل من قدم لك ديكًا روميًا صغيرًا في الاحتياط؟

    وسمعت أيضًا أن Sage وZwicky أنشأا مجموعة على WhatsApp، لذا لا تفوت الفرصة.

  131. يهودا
    أعني أنك تفترض أن الفضاء يمتص الفقاعات التي تنقل الجاذبية، أليس كذلك؟ إنه يتناقض نوعًا ما مع ما نفترضه أن الفضاء لا يمتص أي شيء، أليس كذلك؟
    إذا كنت تعتقد ذلك، يجب عليك وصف آلية الامتصاص.
    عليك أيضًا أن تناقض الأدلة الأخرى الخاصة بالمادة المظلمة.

    بالنجاح….

  132. للمعجزات
    لكل ظاهرة تنتشر في الفضاء على شكل كرة، مثل الجاذبية والموجات الصوتية والموجات الضوئية وغيرها، هناك عاملان يؤثران على الانتشار. و. ما يعرفه الجميع هو التوسع الكروي الذي تتحدث عنه أيضًا بحق، ولكن بالإضافة إلى ذلك هناك اضطراب في الوسط، على سبيل المثال، ينتشر الضوء من المنارة وفقًا لمربع المسافة ولكن أيضًا وفقًا لتعكر الغلاف الجوي. الغيوم التي تبتلع 50 بالمائة كل كيلومتر (بسبب الضباب، على سبيل المثال) ستترك الضوء بالكاد لكل ميل مما هو متوقع بناءً على مربع المسافة.
    أدعي أنه كما هو مذكور أعلاه بالنسبة لأي ظاهرة تنتشر بطريقة تربيعية بما في ذلك الجاذبية، فإن السؤال هو ما هي المسافة التي سيتم التخلي عن 50٪ من الجاذبية. والحال الآن مع نيوتن هو أن هذه المسافة لا نهائية، مما يعني أن كل الجاذبية التي تنطلق تصل إلى وجهتها. أزعم من الحسابات التي أجريتها أن حوالي بضع سنوات ضوئية لن تترك سوى 50% من الجاذبية

    الى ارييل
    ردك الثاني
    حسنًا، قلت إنه من الممكن التعامل مع الجاذبية بدلاً من ذلك، باستخدام الصيغة الدائرية التي تمثل في الواقع قانون نيوتن الثاني F=M*a
    الموجود في الدائرة هو R/F=M*V2
    ولكن هذه هي بالضبط فكرة البروفيسور ميلجروم في نظرية MOND، وقد نفذها دون نجاح كبير. هناك ظواهر تجد هذه النظرية صعوبة في تفسيرها.
    أعني أن ما أفعله بالجاذبية يشبه ما فعله البروفيسور ميلجروم بالقانون الثاني. الاستنتاجات مختلفة.

    الى ارييل
    تعليقك الأول
    أتصور أن نظامًا شمسيًا في مجرة ​​بعيدة يشبه نظامنا الشمسي في الحجم سيتصرف بنفس الطريقة التي يتصرف بها نظامنا الشمسي، أي وفقًا لقوانين نيوتن في الجاذبية للمسافات الصغيرة.
    بالإضافة إلى ذلك، أعتقد أن المجرة البعيدة ستتصرف أيضًا فيما يتعلق بمسافات كبيرة تصل إلى آلاف وعشرات الآلاف من السنين الضوئية مثل مجرتنا، لكن كلاهما لن يفعل ذلك وفقًا لقوانين نيوتن في الجاذبية، بل وفقًا لقوانين أخرى أو منقحة. .
    لا يمكن للمبدأ الكوني أن يزيد نطاق إثبات الصيغ، فقط التجربة والقياسات يمكنها فعل ذلك!

    يرجى الرد بلطف
    سابدارمش يهودا

  133. يهودا
    وراء معادلة الجاذبية هناك منطق بسيط للغاية - تعتمد منطقة تأثير مصدر الإشعاع المتجانس على مساحة الكرة، والتي تعتمد على مربع نصف القطر. إذا نظرنا إلى الضوء، فإن شدة الإشعاع تعتمد على عدد الفوتونات لكل وحدة مساحة لكل وحدة زمنية.
    هل تعتقد أن هذا خطأ أيضًا؟

  134. يهودا، إذا ألقيت المبدأ الكوني في سلة المهملات (وهو ما فعلته في ردك الأخير) فلا يزال يتعين عليك الإجابة على الأسئلة التالية:
    1) ما هي قوانين الحركة التي تعمل في المجرات البعيدة؟ (بعد كل شيء، قوانين نيوتن للحركة صالحة فقط في 10 وحدات فلكية)
    2) إذا لم يكن هناك تناحيم، فلماذا تكون درجة حرارة إشعاع الخلفية وكثافة المجرات واحدة؟
    3) هل المجرة البعيدة التي ألاحظها في هذه اللحظة تتصرف حسب القوى الموجودة في منطقتي أم حسب القوى الموجودة في منطقتها؟ على سبيل المثال، إذا كانت الجاذبية أقوى بعشر مرات، فهل تؤثر علي قوة أقوى بعشر مرات مما أؤثر عليها؟ أم أننا نؤثر بقوى متساوية على بعضنا البعض، وإذا كان الأمر كذلك فما هي قوة تلك القوة؟

  135. الى ارييل
    لقد قلت: "لا يمكننا أن نستنتج أن شيئًا بحجم النظام الشمسي سيكون صالحًا للكون بأكمله". نهاية الاقتباس.
    هذا صحيح، لكنني لم أقله، فقد قاله الفيلسوف الإنجليزي ديفيد هيوم وغيره: "مما تقيسه، لا تعرف إلا ما تقيسه". على سبيل المثال، راجعت 1000 كتاب ورأيت أنها مصنوعة من الورق وهي تحكي قصة بينوكيو، لذلك لا يمكن أن يكون الاستنتاج أن كل الكتب في الكون مصنوعة من الورق وتتعامل مع بينوكيو، ولكن فقط هناك في أقل من 1000 كتاب مصنوع من الورق في الكون ويتعامل مع بينوكيو. وأكثر من ذلك، إذا كان هناك شيء صحيح في النظام الشمسي لعشر وحدات فلكية من الطول، فليس من الضروري مطلقًا أن يكون صحيحًا لمليون أو مليار طول، أي.... إذا كانت الأرض تدور حول الشمس بسبب بالنسبة للجاذبية، هذا لا يعني أن المجرات الحلزونية التي هي أكبر بمليار مرة تدور أيضًا بسبب الجاذبية. ومن فضلك لا تقتبس لي المبدأ الكوني لأن ما يقوله المبدأ هو أنه في نظام شمسي آخر مشابه لنظامنا، تعمل الجاذبية. ما يجب القيام به ولكن هذا ما هو موجود!، آسف.
    أرجو الرد بلطف، إنه مجرد علم (:))
    سابدارمش يهودا

  136. يهودا لماذا حددت الجاذبية؟ لماذا لا تصف معادلاتنا الحركة الدائرية؟ لماذا لا ندع القوة الكهرومغناطيسية تبدأ في التأثير على الجسيمات المحايدة أيضًا؟ وفقا لمنطقك، فإن كل القوى التي اكتشفناها ودرسناها في النظام الشمسي يمكن أن تكون هي العوامل، كما قلت، لا يمكننا أن نستنتج أن شيئا بحجم النظام الشمسي سيكون صالحا للكون بأكمله.

  137. يهودا لماذا حددت الجاذبية؟ لماذا لا تصف معادلاتنا الحركة الدائرية؟ لماذا لا ندع القوة الكهرومغناطيسية تبدأ في التأثير على الجسيمات المحايدة أيضًا؟ وفقا لمنطقك، فإن كل القوى التي اكتشفناها ودرسناها في النظام الشمسي يمكن أن تكون هي العوامل، كما قلت، لا يمكننا أن نستنتج أن شيئا بحجم النظام الشمسي سيكون صالحا للكون بأكمله.

  138. يهودا، من هو العبقري "الموهوب" الذي صور محاضرتك على اليوتيوب؟ الجودة فظيعة بكل بساطة، وسأشعر بالخجل من وضع شيء كهذا على الإنترنت.

  139. للموظف
    بادئ ذي بدء، أنا أستبعد الجاذبية بشكل كامل تقريبًا على المسافات الكونية الكبيرة. ولم يتم إثباته مطلقًا على مسافات كونية كبيرة. اكتشف نيوتن الجاذبية وادعى أنها محددة على الكون كله ومنذ ذلك الحين وهي كذلك. لكن كون نيوتن كان صغيرا، فقط النظام الشمسي إلى كوكب زحل، حوالي عشر وحدات فلكية. فإذا جاز القول أن الصيغة تعمل على الكون كله. ويجب ألا نفترض ذلك لملايين ومليارات السنين الضوئية.
    وفيما يتعلق بفرق الضغط، دعونا نقتل بقرة مقدسة أخرى ونسأل أنفسنا هل كوننا فارغ أم أنه يحتوي على عدد لا يحصى من الجسيمات التي تتحرك داخله، مثل النيوترينوات والأشعة الكونية الأخرى والفوتونات وغيرها. كل هذه العناصر تحدد الغاز، مما يعني أن كوننا عبارة عن جسم غازي ضخم يحتوي على نجوم. والغاز له ضغوط، واختلافات في الضغط، ورياح، وما إلى ذلك. وزعمت أنه بعد استبعاد جميع الاحتمالات الأخرى، بقي حل فرق الضغط فقط في صراع مع فكرة الكتلة المظلمة. وبالطبع يجب ألا ننسى الفراغ والطاقة المظلمة والتوسع المتسارع للكون، كل شيء يحتاج إلى تفسير.
    هذا كل شيء باختصار، وإذا كنت تريد شرحًا أكثر شمولاً، فحاول أن تجدني وتتصل بي وسأكون سعيدًا بلقائي وشرح أفكاري لك وللآخرين، عليك فقط أن تأتي بعقل متفتح وبدون تحيزات. على سبيل المثال، إذا قلت: "والحقيقة هي أنني لا أستطيع رؤية حل لا يتضمن تأثيرات الجاذبية،" نهاية الاقتباس، فهذا بالفعل قرار يجب أن تكون على استعداد للتخلي عنه.
    يرجى الرد بلطف
    سابدارمش يهودا

  140. يهودا، لقد شاهدت الفيديو ولم أفهم حقًا ماذا تعني نظريتك "مقارنة الضغوط"؟ هذا لا يعني الكثير.. ربما يمكنك أن تشرح أبعد من ذلك.. إلا أنني لا أفهم حقًا.. هل أنت في الواقع تدحض نظرية الجاذبية؟ هل هذا ما تقوله لأنه يبدو لي أنه يجب عليك تحسين ما قلته هناك في العرض قليلاً، فأنا شخصياً لا أعرف الكثير عن مشكلة الكتلة المظلمة باستثناء حقيقة أن هناك تناقضاً في المعادلات التي تنشأ من معادلة الحركة ( لكن ما أعرفه أطول قليلاً من المشكلة كما عرضتها بشكل مبسط...) والحقيقة ليست أنني أستطيع رؤية حل لا يتضمن تأثيرات الجاذبية، بشكل عام ما تقوله هناك غير واضح بعض الشيء، إذا ربما يجب عليك الاتصال بمكان يقدم ما تتحدث عنه بشكل أكثر أكاديمية ورسوخًا.. شكرًا

  141. بالنسبة لأولئك المهتمين بشرح قصير، فيما يلي إشارة إلى عرض الـ 14 دقيقة

    https://www.youtube.com/watch?v=kAo5BQQpBqQ
    وبالإضافة إلى ذلك، هناك إشارة إلى محاضرة أكثر شمولاً ألقيتها في الجمعية الفلكية الإسرائيلية
    يوم جيد
    وكم هو ممتع أنهم اخترعوا مكيف الهواء!
    يهودا

  142. ولم أفهم سبب وجود التناقض الواضح. ومن يعرف مقالا علميا يشرح التناقض فأنا أحب أن أطلع عليه.

    من يريد مشاهدة شرح كارول عن تيباس وعنقود الرصاص ولماذا اللعب بالجاذبية بدون المادة المظلمة لا يستطيع شرح المعلومات التي وصلت فهو مدعو للمشاهدة ابتداء من الدقيقة الخمسين على اليوتيوب التالي
    (لقد أحضرتها من قبل ولكن يهودا يصر على عدم الرؤية أو الفهم...)
    http://youtu.be/iu7LDGhSi1A

  143. أقوم بإرفاق رابط للنظرية الأصلية
    منشور منذ 12 عاماً..

    فهو يحل الحاجة إلى المادة المظلمة ...

    سيكون عليك فقط التكيف مع ما يسمى بالعقلية
    "ثقب أسود فائق الضخامة" قادر على بعث مادة...

    صدفة
    لقد تغيرت العديد من روابط الصور الخارجية على مدار 12 عامًا

    http://www.mish-pat.com/assil2/assil2.htm

  144. السيد / السيدة لا يريد العودة إلى القرن السادس عشر
    ماذا ستفعل في القرن السادس عشر؟ نيوتن لم يولد بعد، وجاليليو لم يبني تلسكوبًا بعد. ربما ستتمكن من رؤية جيوردانو برونو وهو يحصل على الحصة ولكن لست متأكدًا. كما أنها ليست مسرحية مؤثرة تستحق العودة إلى القرن السادس عشر.
    علاوة على ذلك، لو كنت هناك في ذلك الوقت، فلا شك أنني كنت سأكون هادئًا جدًا وحاولت عدم إزعاج محاكم التفتيش حتى لا تُثقب مؤخرتي بالطاقات العالية التي كان من الممكن أن ينظمها لي شخص ما. .
    ويجب أن أشير إلى أن استمرار تعليقك يهمني أيضًا، وموضوع النيوترينو وسرعته كثيرًا ما يزعجني. لقد انفجر المستعر الأعظم 1987A في سحابة ماجلان الكبرى منذ 170,000 ألف سنة، ونرى أنها معجزة أن تصل إلينا نيوترونات الانفجار وضوء الانفجار معًا تقريبًا، وهذا على ما يبدو تناقض في النسبية!
    لذلك كان لدينا شيء لنتحدث عنه إذا التقينا ولم يتدخل ألبينزو. ربما كنا سندعوه أيضاً؟...والروضة إستر ماجن بامبي؟؟.
    كان يمكن أن يكون لطيفا.
    مساء الخير
    يهودا

  145. وفي الوقت الحالي نُشر على الموقع الإلكتروني خبر وفاة الفائز بجائزة إسرائيل البروفيسور يعقوب بيكشتاين. سمعته يحاضر عن الكتلة المظلمة ونظرية موند. كما تمت مقابلته عدة مرات حول هذا الموضوع. وكان من أعظم الباحثين في إسرائيل.
    حزين.

  146. يهودا،
    لكنك تسمع من علماء الفيزياء، بما في ذلك أولئك الذين لم تتجادل معهم بعد، أن هناك أدلة دامغة على أنه بغض النظر عن كيفية اللعب مع الجاذبية، هناك نتائج لا يمكن تفسيرها بمثل هذه اللعبة. تشرح كارول ذلك على خلفية النتائج التي توصلت إليها مجموعة Bullet Cluster، بشكل واضح وبلغة إنجليزية سهلة، على موقع YouTube الذي أحضرته، وأنت وحدك. أنا حقا لا أستطيع أن أفهم سبب إصرارك. على أساس ما هي المعرفة التي تتعارض مع الادعاء الذي قدمه كارول؟ بناء على ما هي النتائج؟

  147. ارييل
    لقد أوضحت وجهة نظرك جيداً ودعونا لا ننسى أن ألبانزو في النهاية هو الحكم وهو صاحب القرار وله اتصالات في اتحاد الحكام. هذا ما هو موجود.

    شموليك
    وفي المجرات الحلزونية، قارنوا الجاذبية بقوة الطرد المركزي واكتشفوا أن قوة الطرد المركزي تبلغ عشرة أضعاف قوة الجاذبية. وبما أنه من المفترض أن يكونا متساويين بينهما، أضف الكتلة المظلمة. وعظمة البروفيسور ميلجروم تكمن في أنه أخذ البيانات الموجودة المقاسة ميدانيا، ولم يغيرها، ولكنه قام بتغيير صيغة القوة الطاردة المركزية - قانون نيوتن الثاني - قانون التسارع F=m*a بحيث أصبحت القوة الطاردة المركزية سيكون أصغر بعشر مرات وبعد ذلك سيكون هناك مساواة. بمجرد فشلها. الحل المتبقي هو العمل في اتجاه تغيير صيغة الجاذبية.
    مساء الخير وكم هو ممتع وجود مكيف في الغرفة!
    يهودا

  148. هناك فكرة متكررة مثيرة للاهتمام لدى جميع منكري العلم من أي نوع.
    هذه الحجة - لقد اختبرتها فقط لمسافات أو أوقات قصيرة، وبالتالي قد تكون خاطئة...
    من المفترض أن تكون قوانين الفيزياء عالمية. نفس الحقول/القوى بمعادلة واحدة تصفها.
    فقط لأنني لم أزر هونولولو، لا يعني أنني لا أستطيع أن أفترض أن نفس قوانين الفيزياء تنطبق هناك.
    يبدو لي أن هذا هو ما يقوم عليه فرع علم الكونيات بأكمله.
    وعلى الرغم من كل ذلك... يجب أن أعترف أن صبرامش هو فيلم ترفيهي خفيف يستحق المشاهدة. ولدي أيضًا كل أنواع الأفكار الهرطقة أحيانًا حول وجود بوزون هيغز. والنيوترينوات أسرع من سرعة الضوء.
    من فضلك لا ترد بلطف.

  149. يهودا، قد تكون على حق وقد لا تكون كذلك، لكن في هذه الأثناء لا تنس أنك في المدرجات والحكم ألبينزو في الملعب، قد ترى خطأ تحكيمي أو "خطأ" وألبينزو لا لا تطلق الصافرة، لكن لا تنس أن ألبينزو هو من هو الموجود في الملعب وليس أنت، فأنت لا تعرف ما إذا كان العشب مبللاً أم لا يتم قصه، وما إلى ذلك... وألبينزو يعرف بالفعل. بالطبع، يمكنك الحصول على هذه البيانات ومن ثم تحديد ما إذا كان هناك "خطأ" أم لا، ولكن حتى تحصل عليها، لا يمكنك الطعن في قرارات الحكم. بالإضافة إلى ذلك، أنت أيضًا من مشجعي نفس الفريق، وبالتالي فإن حكمك متحيز مقارنة بالحكم الأكثر حيادية. آمل أن أكون قد أوضحت وجهة نظري.

  150. يهودا،
    MOND ليس هو السبب الذي أوصيك بالتحدث معه (مرة أخرى، في حالتك) ولكنه سيحاول أن يشرح لك لماذا لا يقوم أحد بتغيير البيانات (بمعنى أنه لا أحد يغير البيانات التي تأتي من تلسكوباتنا، وما إلى ذلك). لا أحد يعرف ما إذا كان MOND خاطئًا، ولكن حتى لو كان جيدًا، فلا يمكن حله عن طريق المادة المظلمة.

    بالإضافة إلى ذلك، من المؤسف أنك توقفت عن الاستماع لأنه في المحاضرة الثانية، في حوالي 40 دقيقة، يشرح لماذا تتحدى Bullet Cluster إلى حد العبث أي نظرية تحاول تفسير النتائج فقط من خلال اللعب بالجاذبية. من المؤسف أنك أتيت إلى الحوض الصغير ورفضت أن تشرب منه. جدد طاقاتك المظلمة المفقودة

  151. إلى شموليك
    منذ سنوات مضت، أتيحت لي الفرصة للتحدث مع البروفيسور ميلجروم حول نظرية MOND. اجتماع مثير للاهتمام. لسوء الحظ، فإنه لا يجتاز اختبار الإثبات. على سبيل المثال، فهو غير قادر على تفسير سلوك المجرة M94، التي لا تحتوي تقريبًا على مادة مظلمة.

  152. إلى شموليك
    دخلت إلى الرابط الذي أرسلتني إليه وأدركت أن المحاضر كان يحاول إقناعي بأن نظرية MOND ليست حلاً لما نسميه مشكلة الكتلة المظلمة وبما أنني أتفق مع السيد المحاضر لم أجد أنه من المناسب أن مواصلة الاستماع إلى محاضرته. في هذه الأيام المضطربة التي تجاوزت فيها مرحلة روضة الأطفال "بامبي"، يجب أن أستفيد على أفضل وجه من الطاقات العالية القليلة المتبقية لدي.
    وللعلم، إلى جانب نظرية MOND وفكرة المادة المظلمة، هناك ما لا يقل عن عشرة خيارات أخرى لحل المشكلة الكونية المذكورة أعلاه. شكرا على أي حال.
    يرجى الرد بلطف
    يهودا

  153. الانتظار ربما لأنني كتبت بريد ميلجروم الإلكتروني (من موقعه على الإنترنت في معهد وايزمان) حتى يتمكن يهودا من سؤاله مباشرة عن المادة المظلمة

  154. يهودا،
    هل شاهدت المحاضرات التي أحالتك إليها؟
    الموقف الذي تصفه: "من الواضح أنك لا تفهم أن المادة المظلمة هي تغيير البيانات الذي تم إجراؤه لمساعدة المادة العادية على إكمال الـ 90 بالمائة التي تفتقر إليها لإكمال الجاذبية التي تفتقر إليها" هو ببساطة افتراء على جميع المشاركين في هذا الأمر تقريبًا. الميدان وأنا حقًا لا أفهم سبب مجيئك إلى هذه الأماكن.

    لدي تحدي لك: حنان حصلت لنا على موقع ميلجروم. هنا:
    http://www.weizmann.ac.il/particle/milgrom/
    وهذا هو بريده الإلكتروني: Moti.Milgrom@weizmann.ac.il

    ماذا عن إرسال بريد إلكتروني إليه تشرح فيه ادعاءاتك ضد المادة المظلمة (وإذا كنت شجاعًا فاكتب الجملة التي افتتحت بها مشاركتك الأخيرة)؟ إجابته ستكون مثيرة للاهتمام للغاية

  155. إلى ألبينزو
    طلبت في القسم 3 من ردك ما يلي:
    "أعط مثالاً واحدًا لفيزيائي يغير باستمرار البيانات المقاسة لدعم المادة المظلمة." نهاية الاقتباس.
    من الواضح أنك لا تفهم أن المادة المظلمة هي عبارة عن تعديل للبيانات تم إجراؤه لمساعدة المادة الطبيعية على تعويض الـ 90 بالمائة المفقودة منها لتعويض الجاذبية المفقودة. تغيير سيء، أولئك الذين لا يفهمونه، سيئون للغاية! أنا آسف يا صديقي ألبانزو لأنك تنظر بازدراء إلى روضة الأطفال المحبوبة "بامبي" وكلية شنكار. ليست جميلة!

    الى ارييل
    لا سمح الله أن أسيء إلى المعرفة الواسعة التي يتمتع بها ألبينزو فيما يتعلق بالطاقات العالية والمنخفضة و/أو، في نصف الصاري، ما أخبرت ألبينزو أن الأمر يتعلق بـ: أبسط الأشياء في العلم مثل متى يجب استبدال فكرة علمية، وما إذا كانت الصيغة (على سبيل المثال، الجاذبية) قد أثبتت نفسها على مسافة أقل من ألف سنة ضوئية (النظام الشمسي الداخلي) يجب أن تكون صحيحة أيضًا على مسافات ملايين السنين الضوئية؟ هذا ما علمني إياه مدرس رياض الأطفال ديفيد يوم في روضة "بامبي". هذه أشياء أساسية للغاية والمعرفة الكبيرة التي يتباهى بها ألبينزو، معرفته الهائلة بالفيزياء، لا تنتمي إلى الأشياء الأساسية المذكورة أعلاه ولا تضيف إلى فهم الأشياء، بل وربما تنتقص منها. لذلك بالطبع يمكن لأي شخص أن يتحدث عن المادة المظلمة أو روضة "بامبي" المحبوبة، لكن الاستقرار العقلي لأطفال روضة "بامبي" المحبوبة لا يرتبط بحالة الطاقات العالية في الكون الكوني. لذا يرجى أن تكون دقيقًا ولا تقل أنني قلت أنه لا ينبغي عليه ذلك!
    في الفصل الأول في كلية شنكار (أو "جان بامبي" لم أعد أتذكر ذلك جيدًا) قالت معلمة الإدارة (أو معلمة رياض الأطفال؟) أنه يمكنك أن تكون ذكيًا جدًا ومتميزًا جدًا، لكن ضع في اعتبارك أن الشخص من المستوطنة أو الدرجة الأخرى سيأتي دائمًا ويجد أخطائك. إنه يشبه حقًا مشجع كرة القدم في بيتار القدس وهو ينظر من أعلى المدرجات إلى مباراة كرة قدم مع أفضل اللاعبين ويكتشف أخطاء الحكم ألبانزو في مباراة كرة القدم في بيتار القدس. "La Familia" تثبت ذلك في كل مباراة.

    ولكن يا له من يوم جميل وممتع!
    لذلك أنا في مزاج جيد لذا يرجى الرد بلطف ولا تحبطني.
    يهودا.
    فخور بخريجي "بامبي" شنكار وكلية رياض الأطفال.
    من محبي مكابي .

  156. يهودا،
    ردين. بلدي الأصلي
    المزيد عن MOND وTeVes
    هنا حديث لشين كارول حيث يتحدث عن أفكار حول كيفية تجاوز النسبية العامة. في حوالي الدقيقة 31:20 بدأ الحديث عن MOND وTeves وفي حوالي الدقيقة 40 وصل إلى النقطة التي تجعل هاتين الفكرتين تفشلان في التخلص من المادة المظلمة.

    http://youtu.be/SwyTaSt0XxE

  157. "تخرجت من روضة "بامبي" (مع مرتبة الشرف الأولى)، وشاهدت فيلم "Terminator 2"، ولدي خبرة أكثر من 30 عامًا في الاستحمام بنفسي دون مساعدة الأب أو الأم. وفي رأيي المتواضع أن هذه المعرفة كافية وتسمح بتحليل بنية الشبكات العصبية في الدماغ. في الواقع، لا أفهم لماذا لا يُسمح لي بإجراء العمليات الجراحية لمرضى سرطان الجهاز العصبي..."

    ألبانزو، لقد قتلتني...

    يهودا، قد يبدو خطاب ألبانزو مسيئًا للغاية بالنسبة لك، لكن الرسالة التي تنبثق منه دقيقة تمامًا: تعليمك، كما تصفه بصراحة شديدة، غير كافٍ تمامًا - ولكن في الواقع، تمامًا - للتعامل مع الأسئلة التي هي ثلاثة أوامر حجمها أبسط من مسألة المادة المظلمة.

    يحتاج الإنسان إلى معرفة ما هي الأدوات التي يملكها في ترسانته. لن تخوض معركة ضد كتيبة من المظليين المسلحين من الرأس إلى أخمص القدمين والمسلحين بالعصا. لن تحاول حل مشكلة التنبؤ بالطقس عندما تكون مجهزًا بحسابات الصف الخامس. إن موضوع المادة المظلمة يحتاج إلى أعماق وآفاق يستغرق بناؤها سنوات عديدة، ولا توجد طرق مختصرة هنا.

    وكان من المناسب لك أن تشير إلى تعليقات محددة: الجوهر وليس الكتلة. قد يكون للمادة العديد من الخصائص، إحداها الكتلة. نموذج أو نظرية وليس صيغة. في فيزياء القرن الحادي والعشرين، لم يتم "اختراع" الصيغ، ولكن الصيغ المناسبة مشتقة من نماذج أو نظريات مبنية على مبادئ أساسية قوية جدًا وشاملة جدًا. "الصيغة" هي مجرد زاوية أخرى من النموذج. وبعد كل ذلك، إذا كانت الحسابات والتنبؤات التي تم الحصول عليها من النموذج لا تتطابق مع الملاحظات، فسيتم استبدالها أو التخلي عنها على الفور (أو ربما يجب إجراء إعادة معايرة المعلمات أو، لسوء الحظ، تشوه هيكلي) حتى يتم قبول المباراة من قبل مجتمع البحث بأكمله. لا أحد يحاول "تسوية" الأمور، بل على العكس من ذلك: معايير البحث في الفيزياء عالية جدًا.

    ولكن، كما ذكرنا، من أجل إثبات ذلك، عليك أن ترتفع كثيرًا إلى مستوى أعلى بكثير من مستوى الدراسات الثانوية أو الهندسة الصناعية ولن تساعد أي طرق مختصرة. وبدون الإقلاع يبقى الأفق ضيقا دون أن يعلم الرائي أنه ضيق.

  158. يهودا، من ناحية أنت تدعي أن الفيزيائي الذي يتعامل مع الطاقات العالية لا يمكنه التحدث عن الجاذبية على الإطلاق، ولكن بعد فقرات قليلة تذكر أنه يمكنك مناقشة الموضوع لأنك خريج كلية شينكر في الإدارة الصناعية (والتي كما بقدر ما أعرف لا علاقة له بالجاذبية). أين هو ثباتك؟!

  159. وهناك شيء واحد نسيت أن أتناوله - وهو الغرض من المقال. الغرض من هذا المقال هو إخبارك عن ظاهرة جديدة بدأ رؤيتها تجريبيًا، وهي القوى المؤثرة بين جزيئات المادة المظلمة. في هذا الصدد، القوى الموجودة في القطاع المظلم، أو قوى الظلام. هذا هو الهدف، ولتحقيق هذا الهدف هناك أيضًا فقرة أو فقرتان من الخلفية. في قسم الخلفية، يشرحون المادة المظلمة وأيضًا الصعوبات في النظرية. إذا قرأت ما هو مكتوب بدلا من تغطية أذنيك والصراخ، سترى أن كل الصعوبات التي يتم الحديث عنها هي في فهم الخصائص المجهرية للمادة المظلمة. أي أنه على الرغم من أننا نرى المادة المظلمة بتركيزات عالية في الفضاء، إلا أننا لا نملك في متناول اليد جسيمًا واحدًا يمكننا إجراء تجربة عليه ومعرفة كتلته وشحناته والتفاعلات التي يقوم بها وما إلى ذلك. وفي هذا الصدد، تمتلك النيوترينوات أيضًا كتلة لا نعرف بالضبط ما هي. لذلك في رأيك أنه غير موجود، أليس كذلك؟

  160. يهودا،

    وفيما يلي إشارة موجزة نسبيا. قبل ذلك، أود أن أهنئك على قيامك بعمل *مذهل* (!!!، لا يمكن التأكيد على هذا بما فيه الكفاية) وهو تجاهل كل شيء قلته. لا يمكنني تفسير ذلك إلا من خلال حالة حادة بجنون من قمع الواقع.

    1. سبق أن أوضحت لك أنه لا معنى في العلم للانطباع الذي يأتي من الصياغة واختيار الكلمات لشخص أو لآخر، خاصة عندما يكون هذا الشخص صحفيا وليس عالما في هذا المجال. استمر في الاعتماد على "الشعور باليأس" الذي يشعر به المراسل (هل تفهم الفرق بين المراسل والعالم؟ لست متأكدا)، لأنه ليس لديك أي شيء آخر تتمسك به.

    2. هل المعرفة بفيزياء الطاقة العالية لا تنتمي إلى المادة؟ أولًا، أنت لا تعرف ما هي المعرفة التي أملكها. معرفتي لا تقتصر على الطاقات العالية فقط. والحمد لله، أنا أيضًا أعرف وأعمل وأكتب عن النسبية العامة، والتي من المعروف أنها حد للطاقات المنخفضة بشكل عام للجاذبية الكمية. لكن هذا لا يهم، لأنه - كيف يمكنك أن تقرر ما هو مناسب وما هو غير مناسب؟ كيف يمكن لشخص ليس لديه أي فكرة عن نظرية المادة المظلمة، وليس لديه أي فكرة عن الجاذبية (بخلاف اقتباس صيغة تعلمتها في الصف التاسع)، أن يقرر ما هو ذو صلة وما هو غير ذي صلة؟ ستتعلم أولاً هذا المجال، ثم إذا أردت أدعوك لاختباري ومعرفتي، وما إذا كانت ذات صلة أم لا. سأعطيك تلميحًا: أنا فيزيائي يتعامل مع الجاذبية. الجاذبية، الجاذبية، الجاذبية.

    3. أعط مثالاً واحدًا لفيزيائي يغير باستمرار البيانات المقاسة لدعم المادة المظلمة. لا أعرف حقًا ما إذا كنت كاذبًا أم أنك لا تفهم الفرق بين القياس والفكرة التي تأتي إليك في الحلم. لقد شرحت في تعليق سابق - لا أحد يغير البيانات، بل يغيرون النظرية. المادة المظلمة هي نتيجة لتغير في النظرية (بلغتك الطفولية - تغيير الصيغة. ببساطة بدلاً من تغيير أس المسافة في الصيغة كما تريد أن تفعل دون حتى أن تفهم ما تعنيه، قمنا بتغيير مصادر المادة المظلمة مجال الجاذبية في الصيغة).

    4. هل تحتاج إلى "عالم ذو كرات" لكي "يتخلص من فكرة الجاذبية"؟ لا. بحاجة الى شخص غبي حقا. كم هذا غبي سأعطيكم مثالا على ذلك:

    5. "أنا خريج كلية شنكار في الإدارة الصناعية بمرتبة الشرف، وأدير مشروعًا يتعامل مع تحسين المصانع لأكثر من أربعين عامًا بنجاح كبير. وفي رأيي المتواضع أن هذه المعرفة كافية وتسمح بتحليل مشكلة المادة المظلمة."
    هذا. ببساطة ليس هناك ما يمكن إضافته. الرجل الذي هو غبي بما فيه الكفاية ليقول مثل هذا الشيء، وأيضا غبي بما فيه الكفاية بحيث لا يفهم مدى غباءه، هو في الحقيقة لا يستحق المناقشة. أوافق بالتأكيد على أنه يمكنك القطع هنا.

    أوه، انتظر - شيء أخير. أنا خريجة روضة "بامبي" (مع مرتبة الشرف الأولى)، وشاهدت فيلم "Terminator 2"، ولدي خبرة تزيد عن 30 عامًا في الاستحمام بنفسي دون مساعدة الأب أو الأم. وفي رأيي المتواضع أن هذه المعرفة كافية وتسمح بتحليل بنية الشبكات العصبية في الدماغ. في الحقيقة، لا أفهم لماذا لا يسمح لي بإجراء العمليات الجراحية على مرضى سرطان الجهاز العصبي...

  161. يهودا،
    وحاولت أيضًا أن أوضح لكم، بطريقة غير متداخلة، من خلال محاضرة جميلة جدًا لشون كارول وموضوعها "ما وراء النسبية العامة"، لماذا توجد المادة المظلمة. المحاضرة باللغة الإنجليزية سهلة وموضحة بشكل جيد هناك، في الدقائق التي ذكرتها لماذا لم يتمكن ميلجروم من التخلص من المادة المظلمة. هل تكلفت نفسك عناء مشاهدة المحاضرة؟

    لقد سألت ما هو الغرض من المقال وهنا تفشل فشلا ذريعا في فهم النص وزدته بالانتقاء من خلال تقديم الفقرة الأولى كواجهة للمقال. ويل. الفقرة الأولى درامية لأن غرضها محاولة إدخال القارئ في الموضوع وتنتهي بعلامة استفهام، أي أنها تطلب منك مواصلة القراءة.

    هل وصلت للفقرة الثالثة؟ (أو الثاني، اعتمادا على كيفية العد)؟ ويفتتح بالكلمات التالية:
    "أخذ هذا السيناريو الكئيب منعطفًا مفعما بالأمل في أوائل ربيع عام 2015. فقد كشف الباحثون عن أحد أكثر القرائن إثارة للاهتمام منذ سنوات: علامة على وجود قوة جديدة قد تسمح للمادة المظلمة "بالتحدث" مع نفسها. ستساعد هذه الرؤية في تفسير نوع الجسيمات التي ربما تتكون منها المادة المظلمة.

    ماذا عن هذه الفقرة:
    "لقد أثارت هذه الأخبار الفيزيائيين وألهمتهم للبحث عن إجابات. يقول الفيزيائي نيل وينر من جامعة نيويورك، الذي لم يشارك في البحث: "إذا صمد هذا الاكتشاف، فهو أكثر بكثير من مجرد أخبار مثيرة". السيناريو الذي يتضمن الفوتونات المظلمة هو تعديل للمفهوم الأساسي والمقبول للمادة المظلمة باعتبارها تتكون من نوع واحد فقط من الجسيمات، والذي يُطلق عليه عادةً "جسيم ضخم متفاعل ضعيف"، أو WIMP ("ضعيف"، باللغة الإنجليزية ). لكن فكرة أن المادة المظلمة مصحوبة بفوتونات مظلمة وتفاعلات غريبة قد تساعد في حل بعض المشكلات التي أثيرت من خلال وصف المادة المظلمة باستخدام نوع واحد من WIMP، مثل سبب كون مراكز المجرات أقل كثافة من المتوقع.

    وفي بقية المقال فعل الباحثون ما تتوقعه من أي باحث مناسب وهو إعطاء التنازلات المناسبة، حيث أن الأمر لا يتعلق بالدين بل بالعلم وما أجمل أن يقوم كاتب المقال بتضمين هذه الاقتباسات في المقال .

    فماذا كان في المقال وما هو الهدف منه؟ أخبرنا عن أحدث النتائج في هذا المجال والتي تثير اهتمام جميع العاملين في هذا المجال. الفوتونات المظلمة، قوة بين الجسيمات المظلمة وليست جسيمة مظلمة.. ما أجملها. لقد مر وقت طويل منذ أن قرأت مقالًا أحدثني كثيرًا وقادني إلى الويكي. تحياتي لعيدان!

    لذلك لا أفهم كيف تقرأ مثل هذا المقال وكل ما تأخذ منه هو فقرته الأولى التي تنتهي بعلامة استفهام (!) كهدف للمقال.

  162. يهودا
    فهمك للمقالة هو عكس فهمي. المقال متفائل ولا يدعي في أي مكان أن فكرة هامل المادية خاطئة

    يهودا – أسلوبك غير علمي على الإطلاق. هل لديك فرضية أخرى تحل المشاكل التي تحلها فكرة المادة المظلمة؟ أنا لا أعتقد ذلك. أنت تقدم صيغة تناسب بعض النتائج، ولا تفسر الظواهر الأخرى التي تحلها المادة المظلمة (على سبيل المثال، كما أفهمها، اضمحلال الجاذبية).

    وكما قال أليساندرو - إذا كانت صيغتك تتعارض مع النسبية العامة - فسنتخلص منها أيضًا؟

    كن جادا….

  163. إلى ألبينزو
    انظر الفرق بيننا. قرأت المقال وأرى فيه يأس الكاتب الذي يعترف فيه أنه حتى يومنا هذا لا يوجد أي دليل على حقيقة الكتلة المظلمة ويأمل أن يأتي هذا البحث بنوع من الإثبات.
    أنت، من ناحية أخرى، متأكد مسبقًا مسبقًا من أن كل شيء مثبت، والمادة المظلمة هي حقيقة موجودة، وفي الواقع فإن الأشخاص مثل يهودا يربكون رؤوسهم فقط. إذن أخبرني يا ألبانزو، ما هو الهدف من هذا المقال إذا تم إثبات كل شيء في مجال المادة المظلمة؟

    شيء آخر أراه هنا هو أبسط الأشياء في العلم مثل متى يجب استبدال فكرة علمية، وإذا كانت الصيغة (على سبيل المثال الجاذبية) أثبتت نفسها على مسافة أقل من ألف سنة ضوئية (النظام الشمسي الداخلي) يجب أن يكون صحيحًا أيضًا على مسافات ملايين السنين الضوئية. هذه أشياء أساسية للغاية والمعرفة الكبيرة التي تتباهى بها، فمعرفتك الهائلة بالفيزياء، مثلا الطاقات الكبيرة وغيرها، لا تنتمي إلى الأشياء الأساسية المذكورة أعلاه ولا تزيد من فهم الأشياء، بل وربما تنتقص منها.
    لذلك، مع كل تقديري لمعرفتك، فهو خارج الموضوع هنا.

    وعندما أسمي فكرة المادة المظلمة في الكون بأنها فكرة وهمية، أريد ممن يقرأ ردي أن يفهم رأيي فيها، أن يفهم أن ما يتم خلقه هنا ليس كتلة أو مادة. الذي يحدث هنا هو تغيير البيانات المقاسة في الميدان لتتناسب مع الصيغة أو كما تقول "الفكرة العلمية الموجودة"، وهو تغيير لا ينبغي أن يحدث وهو مخالف لأبسط قاعدة في العلم تناقش السؤال: متى يجب التخلص من فكرة علمية سيئة؟

    ألبانزو، تخلص من فكرة الجاذبية، فهي لا تصلح للمسافات الكبيرة.
    وصحيح أنه يجب على المرء أن يكون عالمًا لديه كرات للقيام بمثل هذا الفعل المتمثل في إلغاء الجاذبية بشكل كامل تقريبًا على المسافات الكونية الكبيرة.

    وأخيرا، للأسف، ربما لم أرد عليك بعد الآن. لقد سئمت فقط من موقفك المتغطرس تجاهي، ومن ألفاظك البذيئة وأسمائك المهينة. أنا خريج كلية شانكار في الإدارة الصناعية بمرتبة الشرف، وأدير مشروعًا يتعامل مع تحسين المصانع لأكثر من أربعين عامًا بنجاح كبير. وفي رأيي المتواضع أن هذه المعرفة كافية وتسمح بتحليل مشكلة المادة المظلمة. لا حاجة لمعرفتك الواسعة.

  164. يهودا،
    أحاول أن أفهم... على افتراض أن لديك، في نظرك، نموذجاً بديلاً يشرح الظاهرة المعنية، هل حاولت نشره في مجلة علمية؟ هل حاولت التعاون مع عالم فيزياء يدعم نموذجك ويرغب في مساعدتك في نشر الأشياء؟ لقد كنت تحمل هذه الآراء لسنوات، ولا بد أنك حاولت أن تفعل شيئًا حيال ذلك، أليس كذلك؟

  165. يهودا،

    1. نعم، أنا واحد من القلائل. كما تعلمون، أنا وعدد قليل من الآخرين الذين نجلس هنا بجانبي في ممر قسم فيزياء الطاقة العالية. وبعض علماء الفيزياء من الجامعة القريبة. وبعضها الآخر من الجامعة المجاورة لها. وأولئك الموجودون في بقية الولايات المتحدة، وأولئك الموجودون في أوروبا، وآسيا، وإسرائيل... كما تعلمون، هذه أقلية صغيرة تبلغ 99% من علماء الفيزياء. وحتى الجزء الصغير من علماء الفيزياء الذين يعتقدون أنه لا توجد مادة مظلمة، مثل ميلجروم، لن يتفقوا معك. لأنهم - على عكسك - اهتموا بمعرفة ما يدور حوله. لقد فحصنا الأدلة، والأدلة، والملاحظات، ونحن نعرف النماذج النظرية. يدرك ميلجروم أنه بغض النظر عما هو مكتوب في كتاب لطلاب المدارس الثانوية البالغين من العمر 15 عامًا، فلا توجد صيغ سحرية في الفيزياء. لا يمكنك أن تقرر أنه بدلاً من المسافة إلى الأس سالب اثنين، يمكنك وضع سالب اثنين ونصف. جميع الروابط تنشأ من النموذج، في حالتنا النسبية العامة، وإذا كنت تريد تغيير "الصيغة"، عليك أن تخترع نظرية جديدة تمامًا للجاذبية. وهو يحاول منذ 30 عامًا القيام بذلك، وعلى الرغم من الاحترام الذي أكنه له (الكشف الكامل: أعرفه شخصيًا)، فإن فكرته لا تنجح. لعقود من الزمن ظلت إما ثابتة، أو تعرضت للضرب من خلال الملاحظات التي تظهر ببساطة أنها غير صحيحة (مثل مجموعة الرصاص التي ذكرتها سابقًا، ورأيت أن شون كارول يتحدث عنها أيضًا في المحاضرة التي أشار إليها شموليك).

    2. بالطبع، ليس لديك أي شيء موضوعي لتقوله، فتلجأ إلى كل أنواع الانطباعات والصياغات لمراسل موقع العلوم، وهو ليس فيزيائيًا في هذا المجال على الإطلاق. ومن خلال صيغها تشعر بأنها غير متأكدة من وجود المادة المظلمة. لماذا أهتم العلم لا يتم بحسب "ما هو الانطباع الذي يحصل عليه يهودا سابدارميش من كلام الصحافي الذي يتحدث عن العلم (ولا يفعل ذلك بنفسه)"، بل بحسب الدراسات. الأدلة والملاحظات والتجارب والرياضيات. كما تعلم، كل تلك الأشياء التي تخاف منها أو كسولًا جدًا بحيث لا تحاول تعلمها. بالطبع، من الأفضل أن تغطي أذنيك وتصرخ بصوت عالٍ: "لا توجد كتلة داكنة!"

    3. وإذا كنا قد تحدثنا بالفعل عن "الكتلة المظلمة"، فأنا أتفق معك في أن الأسماء ليست هي المهمة. كما أتمنى ألا تمانع أنه بدلاً من ميكانيكا الكم سنتحدث عن "فكرة الكم"، وبدلاً من النسبية العامة سندرس "النسبية العامة"، وبدلاً من ميكانيكا نيوتن سندرس "نظرية إسحاق في الكون" حركة". من الواضح أن الأسماء ليست مهمة. لكن عندما تخترع لغتك الخاصة، فإنك تبدو كالأحمق. وعلى وجه الدقة - ما هو موجود هو المادة. يمكن أن تكون المادة ضخمة، ويمكن أيضًا أن تكون غير ضخمة. إنها خاصية المادة. المادة مظلمة وليست الكتلة.

    4. إذن أنا أتحدث فقط وليس لدي ما أقوله، أليس كذلك؟ مضحك. هل سمعت من قبل عن "هافوسل في مومو فوسيل"؟ أنتم لا تكتبون شيئاً في تعليقاتكم، باستثناء مناداتي بـ "السيد العالم"، "نور العلم"، "عالم العلماء" كانت هذه بالطبع عبارات ساخرة تهدف إلى التقليل من قدرتي العلمية. ومن ناحية أخرى، قدمت عددًا من الحجج الملموسة التي تتجاهلها بوضوح. على سبيل المثال، أنك لم تكلف نفسك عناء دراسة الأدلة والملاحظات الخاصة بالنظرية التي ترفضها. على سبيل المثال، أن ادعاءاتك غير صحيحة من الناحية الواقعية (تغيير "صيغة" الجاذبية لن يحل المشكلة - بل على العكس من ذلك، سيخلق العديد من المشاكل لأن أي ظاهرة تفسرها الجاذبية سوف يتوقف تفسيرها إذا قمت بتغيير النسبية العامة). على سبيل المثال، ادعاءك بأن الفيزيائيين غيروا البيانات بدلاً من النظرية هو مجرد كذبة كاملة. وكان جوابي مليئا بالحجج. ولكن بما أنك غبي/جبان جدًا بحيث لا تستطيع مواجهتهم، فأنت تدعي أنني لم أقدم أي حجة موضوعية. اوه حسناً.

    5. بالطبع أنت أيضًا كاذب قليلاً، لأنني لم أقل أبدًا أن "أي شخص يعتقد خلاف ذلك هو بشكل واضح جاهل، غبي، جاهل، وما إلى ذلك، وما إلى ذلك". لقد أوضحت عدة مرات أنني أحترم بشدة محاولات الناس للعثور على تعاليم وتفسيرات بديلة لا تتضمن المادة المظلمة. ماذا تفعل إذا فشلوا، في حين أن المادة المظلمة تتراكم المزيد والمزيد من الأدلة على وجودها. وكما قلت من قبل - فليحيا الفارق البسيط بين عالم فيزياء يدرس الموضوع ويحاول إيجاد حل حقيقي للمشكلة، وبين شخص عديم الضمير عمل على الإنترنت من خلال دخول المنتديات العلمية ونشر الأخبار الخاطئة. معلومات كاذبة عن المادة المظلمة كما أخبرتك، أنا شخصيًا أعرف الأشخاص الذين يدرسون التعاليم البديلة وأضمن لك أنهم إذا قرأوا تعليقاتك فسوف يصابون بالصدمة مثلي على الأقل، إن لم يكن أكثر.

    6. هل تعتقد أن الاستنتاجات من معرفتي غير صحيحة؟ كيف علمت بذلك؟ ليس لديك أي فكرة عن معرفتي. على الأقل 5 (أعتقد أقرب إلى 10) حاولت إقناعك بالقراءة عن النظرية التي تعارضها بشدة. لقد أعطيتك روابط لمقالات تمت مراجعتها من قبل النظراء حول النظرية. لقد أعطيتك روابط لأوراق علمية تسرد الأدلة الإيجابية على وجود المادة المظلمة. أوصيت لك الكتب التي تستعرض هذا الموضوع. بالطبع لم تكلف نفسك عناء الدراسة. لذا، إذا لم تكن لديك أي فكرة عما أعرفه عن المادة المظلمة، فكيف تقرر بالضبط أن استنتاجاتي خاطئة؟

    يمكنك الاستياء من أسلوبي. يمكنك احتضان نسيم، الذي يعاملك لسبب ما بقفازات حريرية. لكن الحقيقة المرة هي أنك لا تختلف عن طوفان الرجعيين الدينيين (وفقط المصابين بالفصام وجنون العظمة الذين يعتقدون أن تكنولوجيا الفلاش جاءت من الفضاء الخارجي) الذين سيطروا على الموقع خلال الأشهر القليلة الماضية. جميعكم تعانين من نفس المرض تمامًا - لقد قررتم ما هو الصواب وما هو الخطأ قبل أن تكلفوا أنفسكم عناء فتح كتاب أو إجراء تجربة. كما ترى، ما يفعله ميلجروم هو العلم. أخذ البيانات والقياسات والأدلة ومحاولة بناء نماذج رياضية توفر التنبؤات واختبارها. ما تفعله هو نقيض ذلك. لذا، على الرغم من أنكما تدعمان تفسيرًا للجاذبية لا يتضمن المادة المظلمة، إلا أنكما مختلفان للغاية. إنه عالم، وأنت تحاول إعادتنا إلى القرن السادس عشر. حظا جيدا في ذلك.

  166. يهودا، فيما يتعلق بالمصطلحات، لا يزال بإمكانك أن تستسلم، على الرغم من أنني لا أفهم سبب إصرارك على الحكمة واستخدام مصطلحاتك الشخصية بدلاً من المصطلحات المقبولة لدى الجميع.

    وفيما يتعلق بالأمر نفسه، فقد أوضح لك أن هناك ملاحظات تؤكد بقوة نظرية المادة المظلمة، فلماذا تتجاهلها؟ أو بدلاً من ذلك، كيف يمكنك تفسيرها بشكل أفضل بدون المادة المظلمة؟

  167. يهودا، فيما يتعلق بالمصطلحات، لا يزال بإمكانك أن تستسلم، رغم أنني لا أفهم لماذا تصر على الحكمة وعدم استخدام المصطلحات المقبولة لدى الجميع؟

    لكن فيما يتعلق بالمادة نفسها، فقد أوضح لك أن هناك ملاحظات تعطي تأكيدًا قويًا لنظرية المادة المظلمة، لماذا تتجاهلها أو كيف تفسرها بشكل أفضل بدون المادة المظلمة؟

  168. يهودا، فيما يتعلق بالمصطلحات، لا يزال بإمكانك التخلي عنها، رغم أنني لا أفهم سبب إصرارك على الحكمة وعدم استخدام المصطلحات المقبولة لدى الجميع؟

    لكن فيما يتعلق بالمادة نفسها، فقد أوضح لك أن هناك ملاحظات تعطي تأكيدًا قويًا لنظرية المادة المظلمة، لماذا تتجاهلها أو كيف تفسرها بشكل أفضل بدون المادة المظلمة؟

  169. ليسبري مرنان
    مع كل التقدير الذي أكنه لعلم ألبانزو (ولدي التقدير.... وحتى القليل من الحسد...) أعتقد أن الاستنتاجات من معرفته خاطئة "قليلا" وليس من الضروري أن أتفق معها . وماذا نسمي فكرة الكتلة / المادة / الوهمية / المظلمة / الخيالية / الخيالية / الشفافة / هي في الحقيقة لا يهم طالما أنك تفهم أنها أكبر كارثة علمية في القرن العشرين !!!
    أرجو الرد بلطف، هذا هو العلم!
    سابدارمش يهودا

  170. يهودا، من المضحك بعض الشيء أنه بعد أن قام إلبينتزو بتصحيحك وتوضيح أنه يجب عليك أن تقول "المادة المظلمة" وليس "الكتلة المظلمة"، فإنك تصر على الاستمرار في تسميتها "الكتلة المظلمة".

  171. يونيو،
    أقترح عليك مشاهدة اليوتيوب الذي أرفقته. وهو يشرح بطريقة جيدة ما اكتشفناه تجريبيًا (بما في ذلك ما اكتشفناه بواسطة ميلجروم) وأين يظهر MOND وTeVes في الصورة

  172. יוני
    وأوضح ألبانزو مراراً وتكراراً - أن المادة المظلمة ليست "حلاً" مخترعاً لمشاكل الجاذبية، بل هناك العديد من الملاحظات الإضافية، التي تؤكد وجود المادة المظلمة.

    إن تغيير صيغة الجاذبية لا يجيب على الأنواع الأخرى من الملاحظات، ولا يفسر شيئًا.

  173. لا داعي للإهانة..
    يفكر يهودا مثل النظرية البديلة التي طورها البروفيسور موتي ميلجروم من معهد وايزمان...
    http://www.themarker.com/magazine/1.2208993
    جيل المادة المظلمة بأكمله - ينبع من الاختلاف بين طريقتي حساب كمية الكتلة... وحقيقة أن الجميع ذهب إلى الحل الأول - لا يبرر أنه هو الحل الصحيح فعلا...

  174. المزيد عن MOND وTeVes
    هنا حديث لشين كارول حيث يتحدث عن أفكار حول كيفية تجاوز النسبية العامة. في حوالي الدقيقة 31:20 بدأ الحديث عن MOND وTeves وفي حوالي الدقيقة 40 وصل إلى النقطة التي تجعل هاتين الفكرتين تفشلان في التخلص من المادة المظلمة.

    http://youtu.be/SwyTaSt0XxE

  175. يهودا
    كيف يمكن دحض هذا؟ كل ما عليك فعله هو تغيير الصيغة مرة أخرى.
    ما أقوله هو أنك شرعت في شرح سبب وجود هذه الظواهر الغريبة.. ما تفعله، في رأيي، يعادل أفلاك التدوير والمتشعبات لبطليموس. أضف ما يكفي - سيعمل، وإذا لم ينجح فسنضيف واحدًا آخر.

    وهذا لا يزال لا يفسر أي شيء ...

  176. يهودا

    أنت الوحيد الذي يكتب هنا
    مما يلقي ظلالا من الشك على التحديدات "العلمية".

    لسبب ما، لا يهاجمونك، كما يهاجمونني...

    ربما لأنك حريص جدًا جدًا على عدم الدوس على الثآليل...

  177. إلى ألبينزو
    شيء واحد يمكن أن يقال عنك أنك من القلائل الذين يؤمنون بحقيقة الكتلة المظلمة دون أدنى شك. الجميع، بما في ذلك مؤلفو المقال المعني، لديهم ما يكفي، كاتب المقال يطلق على واقع الكتلة المظلمة "السيناريو الكئيب"، ويبدأ المقال بالعبارة:
    "هناك شيء ما في مكان ما في الكون. لا يمكننا رؤيته، ولا يمكننا لمسه، ونعلم بوجوده فقط بسبب قوة الجاذبية التي تمارسها على الأجسام الموجودة في الكون. لعقود من الزمن، كانت قصة المادة المظلمة عبارة عن اكتشاف تلو الآخر حول ماهية هذه المادة الغامضة، وهي تصفية تدريجية للاحتمالات جعلت علماء الفيزياء يشعرون بالتوتر المتزايد. ماذا سيحدث عند حذف آخر مرشح من القائمة؟ هل سيكون مصيرنا محتومًا، ولن نتمكن أبدًا من إلقاء نظرة على طبيعة المادة التي تساهم بحوالي 25٪ من الكتلة الإجمالية في الكون؟ نهاية الاقتباس.
    ويبدو لي أن كاتب المقال يدرك أيضاً أن ثمانين عاماً من البحث أكثر من كافية وينهي المقال بعبارة "في رأيي إما الآن أو أبداً".
    لكن بالنسبة لك يا سيد ألبانزو العالم، نور العلم، عالم العلماء، ليس هناك نقص، الكتلة المظلمة موجودة، بلا شك بسبب كل أنواع الكلمات الرنانة التي وضعتها في ردك. ومن يعتقد خلاف ذلك فمن الواضح أنه جاهل وغبي وجاهل وما إلى ذلك وما إلى ذلك. سامحني، لكنك ضائع. ابدأ بالتفكير خارج الصندوق وقد تصل إلى استنتاجاتي.
    وللأسف، وبعيداً عن التقليل من ردي والتشهير به، فإنك لم تقل شيئاً عن الشكوك الموجودة حول موضوع الكتلة المظلمة منذ 80 عاماً. ليس لديك أي شك. حبل!

    للمعجزات
    لماذا تقولون أن معادلة الجاذبية لا يمكن دحضها؟ ولمعلوماتكم تم دحضها عن طريق اختبارها في المجرات الحلزونية والعناقيد المجرية. فقط لأن شخصًا ما قرر تغيير البيانات بالكتلة الوهمية لا يعني أنه لم يتم دحضها.
    فقط، هذا رأيي!
    يرجى الرد بلطف.
    سابدارمش يهودا

  178. يهودا
    سأخبرك كيف أرى ذلك. تنظر إلى القياسات، وتضع صيغة مناسبة لها، ثم تعود إلى المنزل.

    ما هذا والعلم؟ لا يوجد علم هنا، فهو لا يفسر أي شيء، ولا يتنبأ بأي شيء، ولا يمكن دحضه.

  179. متى يستيقظ العلماء فيفعلون بما تقول؟ متى ستتوقف الجامعات عن تدريس الفيزياء. عندما يقرر العلماء أنه ليس من المناسب لهم دراسة المادة التي يبحثون عنها، فإنه يكفي أن نقرأ كتابًا إثرائيًا لطلاب المدارس الثانوية ونبني عليه كل معارفنا. باختصار، عندما يبدأ جميع الفيزيائيين في أن يكونوا أغبياء حقًا.

    كالعادة، يهودا يكتب كومة من الهراء. ولكن هناك هراء واحد يجب معالجته على انفراد، وهناك شيء واحد يكتبه وهو في الواقع ليس هراء.

    الهراء الخاص هو أنه "يكفي أن نبحث عن الكتلة الوهمية". ومن هذا البيان غير المنطقي، ولا سيما من عبارة "الكتلة الوهمية"، يترتب على ذلك أنها غير موجودة. فكيف نخبرك بهذا، لقد وجدناها بالفعل. تم العثور بالفعل على المادة المظلمة (يكفي أن نقول "الكتلة المظلمة" ، ولا أحد باستثناء يهودا سابدارميش يسمي المادة المظلمة بهذه الطريقة). هناك مشاهدات له في جميع أنواع الأماكن في الفضاء. ما لا يزال مفقودًا هو قطعة صغيرة منه، هنا، على الأرض، مما يسمح لنا بدراسة خصائصه المجهرية (مثل تركيبه، والشحنات تحت قوى مختلفة، وما إلى ذلك). لكن المادة المظلمة في التجمعات الكبيرة تمت ملاحظتها لسنوات. يمكن رؤية مثال جميل، على سبيل المثال، في مجموعة التعداد النقطي.

    الشيء الصحيح الذي قاله يهودا هو أنه عندما لا يتناسب النموذج الرياضي مع القياسات، يجب التخلص من النموذج (من فضلك توقف عن استخدام كلمة "صيغة"، منذ مئات السنين لم تكن هناك "صيغ" في الفيزياء. هناك صيغ رياضية النماذج التي تنشأ منها علاقات معينة، ولكن من المستحيل تغيير العلاقات دون تغيير النموذج). وفي الواقع - كان هناك نموذج يحمل أسماء عديدة، على سبيل المثال FRW، وكانوا يعتقدون أنه ممتاز. ثم تابعوا أبعد قليلا، ووجدوا أنه لا يتناسب مع القياسات. النموذج - الذي لم يكن لديه مادة مظلمة أو طاقة مظلمة - ببساطة لا يتفق مع الواقع. لذلك ألقوا بها بعيدا. نظرت ونظرت ووجدت نموذجًا جديدًا. النموذج الجديد يطابق القياسات بشكل خيالي. يحتوي النموذج الجديد على تنبؤات تم تعديلها في المختبر. وكان النموذج الجديد دليلا إيجابيا على صحته. كان النموذج يسمى LCDM، أو باللغة العبرية - "Lamda CDM". Lamda هو الحرف اليوناني الذي يمثل الطاقة المظلمة، وCDM تعني المادة المظلمة الباردة. الجواب، المادة المظلمة الباردة. لكن يهودا يريد منا أن نتخلص من هذا النموذج. لماذا؟ لأنه لا يفهمه... في الواقع، إنه خائف منه لدرجة أنه في كل مرة يتم جلب رابط لمراجعة النموذج، بما في ذلك دليل صحته (مع القياسات - ليس شيئًا نظريًا ولكن تم التحقق منها في المختبر)، فهو يرفض قراءتها بعذر واهٍ ("إنها باللغة الإنجليزية"، "إنها طويلة"، "ليس لدي وقت"...).

    يهودا، هل تريد منهم أن يستجيبوا لك بلطف؟ توقف لسنوات وسنوات عن كتابة نفس التعليق الغبي الذي ينبع من الجهل (تتحدث عن أشياء ليس لديك أي فكرة عنها وترفض صراحة أن تتعلمها) وتفتري على الأشخاص الذين هم أذكى منك بـ 10 مرات، وأكثر صدقًا منك بـ 100 مرة، وأكثر اجتهادًا منك بألف مرة، يا من تحاول فعليًا تعزيز فهمنا للكون من حولنا وعدم إعادته إلى العصور الوسطى.

  180. في الدرس الأول في الثانوية العامة في الفيزياء قال المعلم أنه إذا كانت النتائج التي تم الحصول عليها من الصيغة لا تتطابق مع البيانات المقاسة في الميدان فمن الذي يجب التخلص منه النتائج المقاسة في الميدان أم الصيغة؟ واضح للجميع أنه يجب على المرء أن يفعل شيئًا بسيطًا وهو التخلص من الصيغة والبحث عن صيغة أخرى. الأمر واضح للجميع، لكن في بعض الأحيان تحتاج إلى القليل من الشجاعة. متى سيقف العلماء الشجعان ويقررون أن هذه هي صيغة الجاذبية التي استنفدت نفسها وبعد مرور 350 عاما على أيام نيوتن يجب استبدالها بأخرى. يجب تفسير الحركة في المجرات دون الجاذبية المقدسة!، فلا معنى للتمسك بها، وتغيير المعطيات بمواد وهمية مثل المادة المظلمة والطاقة التي وظيفتها كلها الحفاظ على صيغة الجاذبية بشكلها. لقد مرت ثمانين عامًا منذ أن جاء فريتز زفيكي بفكرة الكتلة المظلمة للحفاظ على صيغة نيوتن-آينشتاين للجاذبية حتى على المسافات الكبيرة بين المجرات وعناقيد المجرات. يكفي أن نبحث عن الكتلة الوهمية. حان الوقت للتغير.
    يرجى الرد بلطف!
    سابدارمش يهودا

ترك الرد

لن يتم نشر البريد الإلكتروني. الحقول الإلزامية مشار إليها *

يستخدم هذا الموقع Akismat لمنع الرسائل غير المرغوب فيها. انقر هنا لمعرفة كيفية معالجة بيانات الرد الخاصة بك.